Você está na página 1de 101

NBME 11

04/07/2014

BLOCK 1
1.FF since TB is a reportable disease
2.CC
When blood sugar is low, glucagon signals the adipocytes to activate
hormone-sensitive lipase, and to convert triglycerides into free fatty acids
3.EE
Ninety percent of patients with gout develop excess urate stores due to an
inability to excrete sufficient amounts of normally produced uric acid in the
urine (underexcretion). The remaining patients either overconsume purines
or produce excessive amounts of uric acid endogenously (overproduction).
In rare cases, overproduction of uric acid is primary, due to a genetic
disorder. These disorders include hypoxanthine-guanine
phosphoribosyltransferase deficiency (Lesch-Nyhan syndrome), glucose-6phosphatase deficiency (von Gierke disease), fructose 1-phosphate aldolase
deficiency, and PP-ribose-P synthetase variants.
Overproduction of uric acid may also occur in disorders that cause high cell
turnover with release of purines, such as meat intake and seafood intake.
These disorders include myeloproliferative and lymphoproliferative disorders,
psoriasis, chemotherapy (tissue lysis), hemolytic anemias, pernicious
anemia, ineffective erythropoiesis (as in B-12 deficiency), excessive
exercise, and obesity.
Overproduction of uric acid can occur from overconsumption of foods high
in purines.

Common causes of secondary gout due to under excretion of uric acid


include renal insufficiency, lead nephropathy (saturnine gout), starvation or
dehydration, hypothyroidism, hyperparathyroidism, drugs (including loop
and thiazide diuretics and cyclosporine A), low-dose aspirin, and chronic
ethanol (especially beer and hard liquor) abuse. These disorders should be
identified and corrected, if possible.
Comorbidities, including hypertension, diabetes, renal insufficiency,
hypertriglyceridemia, hypercholesterolemia, diabetes, obesity, and early
menopause, are associated with a higher incidence of gout.
Consumption of fructose-rich foods and beverages are associated with an
increased risk of gout in both men and women
Because of no clinical symptoms or history we should think about risk factors
that can cause Gout...
....gender...Men more than women
.......................age.....elderly more than younger
4.CC
Gardenella vaginalis....gram variable rod...cause vaginosis
white/gray vaginal discharge
Whiff test...add KOH...fishy smell
non painful
anaerobe
overgrowth of bacteria in vagina
Clue cells or vaginal epithelial cell covered with bacteria
treatment....Metronidazole
5.AA
Check 1904 Q in UW
6.CC
Lovastatin....-statins
HMG-CoA reductase inhibitors...dec. LDL.. Bad cholesterol...mild effect on
HDL and triglycerides...inc.
mechanism of action....
1.inhibit cholesterol precursor , mevalonate
2.synthesizing LDL receptors

7.CC

2-Naphthylamine is an aromatic amine. It is used to make azo dyes. It is a


known human carcinogen.
Aniline dyes...aromatic amines
2-Naphthylamine is found in cigarette smoke and suspected to contribute to
the development of transitional cell carcinoma of urinary tract system
(It is activated in the liver but quickly deactivated by conjugation to
glucouronic acid. In the bladder, glucouronidase re-activates it by
deconjugation, which leads to the development of bladder cancer.)
8.DD
Presentation
Pin-point pupils, severe respiratory depression/cyanosis and coma.
BP may be low but is often surprisingly well maintained and with
pentazocine overdose actually increases.
Although some opiates, e.g. dextropropoxyphene and pethidine, increase
muscle tone and cause fits, in general opiates cause marked hypotonia.

9.CC

Mesothelioma....It is usually caused by exposure to asbestos.


Asbestosis...Ferruginous bodies...Asbestos bodies with prussian blue iron
stain in the lung.
10.DD
plasmid transfer between bacteria needs cell to cell contact through
conjugation. And E coli is notorious for that.

Bacterial transformation may be referred to as a stable genetic change


brought about by the uptake of naked DNA (DNA without associated cells or
proteins) and competence refers to the state of being able to take up
exogenous DNA from the environment. Two forms of competence exist:
natural and artificial.
11.CC

Candida albicans....oral and esophageal thrush in


immunocompromised...neonates,steroides,diabetes,AIDS
and Vulvaginitis.....high PH diabetes,use of antibiotics
Diaper rash and endocarditis in IV drug users, disseminated candidiasis to
any organ,chronic mucocutaneous candidiasis
Treatment....Nystatin for superficial infection( swish and swallow)
Amphotericin B..for serious systemic infection

12.CC
The palmar branch of the median nerve is a branch which arises at the lower
part of the forearm.
It pierces the palmar carpal ligament, and divides into a lateral and a medial
branch;
The lateral branch supplies the skin over the ball of the thumb, and
communicates with the volar branch of the lateral antibrachial cutaneous
nerve.
The medial branch supplies the skin of the palm and communicates with the
palmar cutaneous branch of the ulnar.
Unlike most of the median nerve innervation of the hand, the palmar branch
travels superficial to the Flexor retinaculum of the hand. Therefore, this
portion of the median nerve usually remains functioning during carpal tunnel
syndrome.

13.CC
2,3-Bisphosphoglycerate:
It binds with greater affinity to deoxygenated hemoglobin (e.g. when the red
cell is near respiring tissue) than it does to oxygenated hemoglobin
14.AA
Acute coronary syndrome (ACS) is usually one of three diseases involving
the coronary arteries: ST elevation myocardial infarction (30%), non ST
elevation myocardial infarction (25%), or unstable angina (38%)
The cardinal sign of decreased blood flow to the heart is chest pain
experienced as tightness around the chest and radiating to the left arm and
the left angle of the jaw. This may be associated with diaphoresis
(sweating), nausea and vomiting, as well as shortness of breath. In many
cases, the sensation is "atypical", with pain experienced in different ways or
even being completely absent (which is more likely in female patients and
those with diabetes). Some may report palpitations, anxiety or a sense of
impending doom and a feeling of being acutely ill.
15.AA
Albuterol....Beta2 agonist...relaxes bronchial smooth muscle ....used during
acute exacerbation
...............................................Gs........................................................
..............
Beta1,Beta2,D1,H2,V2 receptors..........Adenylyl cyclase.....ATP..>
inc.cAMP.....>inc.Protein Kinase A
16 AA
17 CC
CMV has a unique method of escaping immune response it dec. the
expression of MHC 1 and also expresses a decoy molecule. The normal for
the MHC 1 molecule is that its made in the RER and then transported to the
Cell membrane. If it get translocated to the cytosol, its likely to be degraded.
Ref Kaplan Immuno Chap 8[pg 99] viral strategies for evasion of the CMI
response

18.GG

Pupillary reflex:
The oculomotor nerve also controls the constriction of the pupils and
thickening of the lens of the eye. This can be tested in two main ways. By
moving a finger toward a person's face to induce accommodation, as well as
his going cross-eyed, his pupils should constrict.
Shining a light into one eye should result in equal constriction of the other
eye. The neurons in the optic nerve decussate in the optic chiasm with some
crossing to the contralateral optic nerve tract. This is the basis of the
"swinging-flashlight test".
19 DD

20.CC

Kernicterus is damage to the brain centers of infants caused by increased


levels of unconjugated bilirubin
21.BB
http://www.helpguide.org/elder/lewy_body_disease.htm
http://www.alz.org/alzheimers_disease_dementia_with_lewy_bodies.a
sp
Lewy bodies are abnormal aggregates of protein that develop inside nerve
cells in Parkinson's disease (PD) and Alzheimer's disease (AD) and some
other disorders. They are identified under the microscope when histology is
performed on the brain.
*Cytoplasmic!*
Parkinson's disease....degenerative disorder of CNS associated with lewy
bodies and depigmentation of the substantia nigra, loss of dopaminergic
neurons
rare cases have been linked with illicit street drug... heroin

22.EE
Hypothyroidism is a condition leading to the deficiency in the production of
thyroid hormone.
Subclinical hypothyroidism is a state of normal thyroid hormone levels,
thyroxine (T4) and triiodothyronine (T3), with mild elevation of thyrotropin,
thyroid-stimulating hormone (TSH).
With higher TSH levels and low free T4 levels, symptoms become more
readily apparent in clinical (or overt) hypothyroidism.
Early....
Poor muscle tone (muscle hypotonia)
Fatigue
Any form of menstrual irregularity and fertility problems
Hyperprolactinemia and galactorrhea
Elevated serum cholesterol
Cold intolerance, increased sensitivity to cold
Constipation
Rapid thoughts
Depression
Muscle cramps and joint pain
Thin, brittle fingernails
Coarse hair
Paleness
Decreased sweating
Dry, itchy skin
Weight gain and water retention
Bradycardia
Late......
Goiter
Slow speech and a hoarse, breaking voice deepening of the voice can also
be noticed, ma.
Dry puffy skin, especially on the face
Thinning of the outer third of the eyebrows
Abnormal menstrual cycles
Low basal body temperature
Thyroid-Related Depression
Uncommon.........

Impaired memory
Impaired cognitive function (brain fog) and inattentiveness.
A slow heart rate with ECG changes including low voltage signals.
Diminished cardiac output and decreased contractility
Reactive (or post-prandial) hypoglycemia
Sluggish reflexes
Hair loss
Anemia caused by impaired hemoglobin synthesis (decreased EPO levels),
impaired intestinal iron and folate absorption or B12 deficiency
Difficulty swallowing
Shortness of breath with a shallow and slow respiratory pattern
Increased need for sleep
Irritability and mood instability
Yellowing of the skin due to impaired conversion of beta-carotene to vitamin
A
Impaired renal function with decreased glomerular filtration rate
Acute psychosis (myxedema madness) (a rare presentation of
hypothyroidism)
Decreased libido in men due to impairment of testicular testosterone
synthesis
Decreased sense of taste and smell (anosmia)
Puffy face, hands and feet (late, less common symptoms)
Gynecomastia
Deafness
23.DD
The palmaris longus is seen as a small tendon between the flexor carpi
radialis and the flexor carpi ulnaris, although it is not always present. The
muscle is absent in about 14 percent of the population
Absence of palmaris does not have any known effect on grip strength
24.AA
Pertussis toxin inhibits Gi, so causes increase in cAMP
-Lymphocytosis promotion
-Hypoglycemia due to release of insulin.
Pertussis Toxin interferes with the early chemokine production and the
inhibition of the neutrophil chemotaxis. Chemokines are signaling molecules
produced by infected cells and attract neutrophils and macrophages.

25.EE
Barrier dysfunction is present in patients with Crohn's disease as well as
some of their healthy first degree relatives. It has therefore been suggested
that increased tight junction permeability (reduced barrier function) is a risk
factor for development of Crohn's disease.
26.AA
Narrowing renal artery inc. Renin-Angiotensin-Aldosterone system.......
27.DD
irreversible cell injury
Plasma membrane damage
lysosomal rupture
Calcium ion influx....oxidative phosphorylation,
nuclear pycnosis,karyolysis,
Mitochondrial pemeability....vacualization and phospholipid-containing
amorphous density
28.BB
The addictive potential of codeine encouraged the marketing of
dextromethorphan in a variety of cough and cold preparations. Although
dextromethorphan is chemically derived from the opiates, it has no analgesic
or addictive properties. The cough suppression potency of dextromethorphan
in adults is nearly equal to that of codeine.2 The drug, like codeine, acts on
the central nervous system to elevate the threshold for coughing.
29.EE
Bone marrow transplant has been shown to induce tolerance following organ
transplant.
30.AA
The patient is too tall and thin with scoliosis.....Marfan's....cystic medial
necrosis
Aortic dissection. Present with tearing chest pain radiating to the back....
CXR.. mediastinal widening
31.DD
adult cell type is replaced by another...often secondary to irritation/or
environmental exposure
Barrett's esophagous....Glandular metaplasia...replacement of non
keratinized squamous epithelium with intestinal/columnar epithelium in the
distal esophagous due to chronic acid reflux. .GERD

32. E
Brain Lymphoma
(PCNSL), also known as micro glioma and primary brain lymphoma, is a
primary intracranial tumor appearing mostly in patients with severe
immunosuppression (typically patients with AIDS). PCNSLs represent around
20% of all cases of lymphomas in HIV infections (other types are Burkitt's
lymphomas and immunoblastic lymphomas).
is highly associated with Epstein-Barr virus (EBV) infection (> 90%) in
immunodeficient patients (such as those with AIDS and those iatrogenically
immunosuppressed), and does not have a predilection for any particular age
group.
usually presents with seizure, headache, cranial nerve findings, altered
mental status, or other focal neurological deficits typical of a mass effect
Systemic symptoms may include fever, night sweats, or weight loss.

Other symptoms include


diplopia
dysphagia
vertigo
monocular vision loss
progressive dementia or stupor in patients with a non focal neurologic
exam and minimal abnormalities on MRI (more common in AIDS patients)
facial hypoesthesia
The definitive diagnosis is arrived at from tissue, i.e. a biopsy, by a
pathologist.
MRI or contrast enhanced CT classically shows multiple ring-enhancing
lesions in the deep white matter.
The major differential diagnosis (based on imaging) is cerebral
toxoplasmosis, which is also prevalent in AIDS patients and also presents
with a ring-enhanced lesion, although toxoplasmosis generally presents with
more lesions and the contrast enhancement is typically more pronounced.
imaging techniques cannot distinguish the two conditions with certainty, and
cannot exclude other diagnoses. Thus, patients undergo a brain biopsy
33. C
The patients treated with the new drug the duration of the outbreak was 1
week, instead that the duration of the patients treated with the placebo was
2 week. It mean that the prevalence in the group treated with the new drug
by half.

34. D

Hemorrhoids -- vascular structures in the anal canal which help with stool
control. They become pathological or piles when swollen or inflamed.
In their physiological state they act as a cushion composed of arteriovenous channels and connective tissue that aid the passage of stool.
The symptoms of pathological hemorrhoids depend on the type
present. Internal hemorrhoids usually present with painless rectal bleeding
while external hemorrhoids present with pain in the area of the anus.
Recommended treatment consists of increasing fiber intake, oral fluids to
maintain hydration, NSAID analgesics, sitz baths, and rest.
Surgery is reserved for those who fail to improve following these
measures.
35. B
Metoclopramide -- antiemetic and gastroprokinetic agent.
Thus it is primarily used to treat nausea and vomiting, and to facilitate
gastric emptying in patients with gastroparesis.
It is also a primary treatment for migraine headaches.
36. E
NNRTI: Nevirapine, Efavirenz, Delavirdine:

Mechanism of action: Preferentially inhibit reverse transcriptase of HIV


, prevent incorporation of DNA copy of viral genome into the host DNA. ( FA
2010 page 195)
37. C
Congenital hearing loss --hearing loss present at birth.
Can be hereditary or due to other factors present either in utero
(prenatal) or at the time of birth.
Treatment:
A child with a congenital hearing loss should begin receiving treatment
before 6 months of age. Studies suggest that children treated this early are
usually able to develop communication skills (using spoken or sign language)
that are as good as those of hearing peers.
In the United States of America, because of a Federal law (the Individuals
with Disabilities Education Act), children with a hearing loss between birth
and 3 years of age have the right to receive interdisciplinary assessment and
early intervention services at little or no cost. After age 3, early intervention
and special education programs are provided through the public school
system.
38. B
Acidosis Respiratory: PO2, PH , CO2
39. A
The formation of granulation tissue in an open wound allows the re
epithelialization phase to take place, as epithelial cells migrate across the
new tissue to form a barrier between the wound and the environment. Basal
keratinocytes from the wound edges and dermal appendages such as hair
follicles, sweat glands and sebacious (oil) glands are the main cells
responsible for the epithelialization phase of wound healing.
They advance in a sheet across the wound site and proliferate at its
edges, ceasing movement when they meet in the middle.
Keratinocytes migrate without first proliferating. Migration can begin as early
as a few hours after wounding. However, epithelial cells require viable tissue
to migrate across, so if the wound is deep it must first be filled with
granulation tissue. Thus the time of onset of migration is variable and may
occur about one day after wounding. Cells on the wound margins proliferate
on the second and third day post-wounding in order to provide more cells for
migration.

40. D
Ropinirole -- non-ergoline dopamine agonist.
-- acts as a D2, D3, and D4 dopamine receptor agonist with highest affinity
for D3.
It is weakly active at the 5-HT2, and 2 receptors and is said to have
virtually no affinity for the 5-HT1, benzodiazepine, GABA, muscarinic, 1,
and -adrenoreceptors.
Ropinirole is metabolized primarily by cytochrome P450 CYP1A2, and at
doses higher than clinical, is also metabolized by CYP3A4. At doses greater
than 24 mg, CYP2D6 may be inhibited, although this has only been tested in
vitro.
41. B

The uterine artery usually arises from the anterior division of the
internal iliac artery
FemoralExternal Iliac Internal Illiac Uterine A.
Uterine artery embolization (UAE) is a procedure where an interventional
radiologist uses a catheter to deliver small particles that block the blood
supply to the uterine body. If the procedure is done for the treatment of
uterine fibroids it is also called uterine fibroid embolization (UFE). Under
local anesthesia a catheter is introduced into the femoral artery at the groin
and advanced under radiographic control into the uterine artery.
42 EE
CMI is an immune response that does not involve antibodies or
complement but rather involves the activation of macrophages, natural killer
cells (NK), antigen-specific cytotoxic T-lymphocytes, and the release of
various cytokines in response to an antigen.

immune
system
separated into
two branches:
humoral
immunity, for
which the
protective
function of
immunization
could be found
in the humor
(cell-free bodily
fluid or serum)
and
cellular
immunity, for
which the
protective
function of
immunization
was
associated with
cells. CD4
cells or helper
T cells provide
protection
against
different
pathogens.
T cells cause
death by
apoptosis without using cytokines, therefore in cell mediated immunity
cytokines are not always present.
Cellular immunity protects the body by:
1. activating antigen-specific cytotoxic T-lymphocytes that are able to induce
apoptosis in body cells displaying epitopes of foreign antigen on their
surface, such as virus-infected cells, cells with intracellular bacteria, and
cancer cells displaying tumor antigens;
2. activating macrophages and natural killer cells, enabling them to destroy
pathogens; and
3. stimulating cells to secrete a variety of cytokines that influence the
function of other cells involved in adaptive immune responses and innate
immune responses.
43. C
Schizophrenia: Periods of psychosis and disturbed behavior with a decline in
functioning lasting > 6 months. Associated with dopaminergic activity,
dendritic branching.
Marijuana use is a risk factor for schizophrenia in teens. It most
commonly manifests as auditory hallucinations, paranoid or bizarre
delusions, or disorganized speech and thinking, and it is accompanied by
significant social or occupational dysfunction.

44. A
Glibenclamide (INN), also known as glyburide (USAN), antidiabetic
sulfonylureas, closely related to sulfa drugs.
Mechanism of action: works by inhibiting ATP-sensitive potassium channels
in pancreatic beta cells.--inhibition causes cell membrane depolarization, -voltage-dependent calcium channels to open, -- an increase in intracellular
calcium in the beta cell, which stimulates insulin release.
45. A
Plasmodium vivax -- protozoal parasite ,a human pathogen. The most
frequent and widely distributed cause of recurring (tertian) malaria
one of the four species of malarial parasite that commonly infect
humans. less virulent than Plasmodium falciparum, which is the deadliest of
the four, and is seldom fatal. carried by the female Anopheles mosquito,
Chloroquine remains the treatment of choice for vivax malaria
32 to 100% of patients will relapse following successful treatment of P.
vivax infection if a radical cure (eradication of liver stages) is not given
Eradication of the liver stages is achieved by giving primaquine, after
checking the patients G6PD status to reduce the risk of haemolysis.
However, in severe G6PD deficiency, primaquine is contraindicated and
should not be used
46.EE
vincristine.....M-phase specific alkaloides that bind to tubulin and block
polymerization of microtubules...so that mitotic spindle can not form
s/e...neurotoxicity....areflexia, peripheral neurities
paralytic ileus
Cyclophosphamide....alkylating agent..covalantly x-link DNA at guanine N-7,
require bio activation by liver
s/e...myelosuppression,hemorrhagic cystitis..can be prevented by mesna
Doxorubicin:
.....generate free radicals and noncovalantly intercalate in DNA
s/e...Cardiotoxicity,myelosuppression and marked alopesia,toxic
extravasation
Rituximab:
General side effects have included fever (49%), chills (32%), asthenia
(16%), headache (14%), and abdominal pain (6%).

47.CC
Perfusion pressure....the gradient between arterial blood pressure and
venous pressure in a comparable location in the vascular tree....in this case
the perfusion pressure is the pressure gradient between renal artery and
renal vein......
relationship between flow(Q) ,Resistance(R),and Perfusion pressure(P1P2)....
According to Poiseuille Equation
P1-P2(Perfusion pressure)= R multiplied by Q.....
P1= Renal artery pressure
P2=Renal Vein pressure
This patient has Hypotension Secondary to Severe Dehydration ....this
causes the activation of the Sympathetic Nervous System ....and the RAA
system ....both work independently even if the end result is the same .....
Sympathetic Nervous System causes Vasoconstriction of arteriolar beds
including the renal arterioles ..... the sympathetic nervous system increases
the resistance and decreases the flow (Q)....Therby decreasing perfusion
pressure ( the difference between arterial presuure and venous pressure)....
The RAA system ....AT II is a potent vasoconstrictor throughout the body
....including the afferent and efferent arterioles .....even if more on
efferent ......When there is vasoconstriction of the afferent arterioles there
will be a decrease in flow (Q)....which further decreases the perfusion
pressure according to Poiseuille equation ....

48.AA
immunologically compromised, esp those with HIV and low CD4 T cell
counts, frequently show negative results from the PPD test. This is because
the immune system needs to be functional to mount a response to the
protein derivative injected under the skin.
With the change from 10mm to 5 mm....there are more people can be
identified with positive tuberculin test.... so incidence and prevalance will be
higher
The results of this test must be interpreted carefully. The person's medical
risk factors determine at which increment (5 mm, 10 mm, or 15 mm) of
induration the result is considered positive.
A positive result indicates TB exposure.
5 mm or more is positive in
HIV-positive person
Recent contacts of TB case
Persons with nodular or fibrotic changes on chest x-ray consistent with old
healed TB
Patients with organ transplants and other immunosuppressed patients
10 mm or more is positive in
Recent arrivals (less than 5 years) from high-prevalence countries
Injection drug users
Residents and employees of high-risk congregate settings (e.g., prisons,
nursing homes, hospitals, homeless shelters, etc.)
Mycobacteriology lab personnel
Persons with clinical conditions that place them at high risk (e.g., diabetes,
prolonged corticosteroid therapy, leukemia, end-stage renal disease, chronic
malabsorption syndromes, low body weight, etc.)
Children less than 4 years of age, or children and adolescents exposed to
adults in high-risk categories
49.CC
The child has less opportunity to use her/his gross muscle for movement and
development....so delayed gross motor development could be occurred
50.AA
Hb electrophoresis---measures the different types of the oxygen-carrying
protein (hemoglobin) in the blood.
Many different types of hemoglobin (Hb) exist. The most common ones are
HbA, HbA2, HbF, HbS, HbC, Hb H, and Hb M.

Healthy adults only have significant levels of HbA and HbA2.


Some people may also have small amounts of HbF (which is the main type of
hemoglobin in an unborn baby's body). Certain diseases are associated with
high HbF levels (when HbF is more than 2% of the total hemoglobin).
HbS is an abnormal form of hemoglobin associated with sickle cell
anemia. In people with this condition, the red blood cells sometimes have a
crescent or sickle shape. The cells easily break down, or can block small
blood vessels.
HbC is an abnormal form of hemoglobin associated with hemolytic anemia.
The symptoms are much milder than they are in sickle cell anemia.
Other, less common, abnormal Hb molecules cause anemias.

Normal Results
In adults, these hemoglobin molecules make up the following percentages of
total hemoglobin:
Hb A: 95% to 98%
Hb A2: 2% to 3%
Hb F: 0.8% to 2%
Hb S: 0%
Hb C: 0%
In infants and children, these hemoglobin molecules make up the following
percentages of total hemoglobin:
Hb F (newborn): 50% to 80%
Hb F (6 months): 8%
Hb F (over 6 months): 1% to 2%
The presence of significant levels of abnormal hemoglobins may indicate:
Hemoglobin C disease
Rare hemoglobinopathy
Sickle cell anemia
Thalassemia
BLOCK 2

1.CC
Aspirin accetylates and irreversibaly inhibits cyclooxygenase..both coc1 and
cox2 to prevent the conversion of arachidonic acid to thromoxane A2....inc.
bleeding time ,no effect on PT and PTT.
2.FF
Levofloxacin is a broad-spectrum antibiotic that is active against both Grampositive and Gram-negative bacteria. It functions by inhibiting DNA gyrase, a
type II topoisomerase, and topoisomerase iv which is an enzyme necessary
to separate replicated DNA, thereby inhibiting cell division.
3.AA
The deltoid is innervated by the axillary nerve. The axillary nerve originates
from the ventral rami of the C5 and C6 cervical nerves, via the superior
trunk, posterior division of the superior trunk, and the posterior cord of the
brachial plexus.

Axillary nerve lesion


***Sensory supply
over deltoid
***Sensory loss
small area over deltoid
***Area of pain
across shoulder tip
***Motor deficit
second 90 of shoulder abduction (deltoid)
(teres minor cannot be evaluated)
***Causative lesions
# neck of humerus, dislocated shoulder
deep IM injection
4.BB

Statistical significance= Is determined by P value.....and as see the p


value is less than .05....hence for sure there is a difference between the two
interventions (NOT BY CHANCE ALONE).....If the P Value is greater than .05
u can SAY the difference in the two interventions is due to CHANCE
ALONE .....So A is out
-Practical Importance means CLINICAL SIGNIFICANCE not Statistical....and
clinical significance MUST ANSWER THE QUESTION ...how effective is the
intervention or treatment in the CLINICAL SETTING....And the doctor made
the CALL in this case not the researcher....And the way u measure CLINICAL
SIGNIFICANCE IS by some of the following methods...EFFECT SIZE ,Number
Needed to treat,and Preventive fraction.....
-Already in the question u stated the magnitude of the difference was so
small....How do they know ? They calculated NNT....
as u know NNT= 1/49%-45 %=1/4%=25.....Taking Program A as a control
(as u know control groups can get placebo or as in this case STANDARD
CARE)....
-This means if the health official decided to change Program B as a
STANDARD of CARE .....to treat HYPERCHOLSTEROLEMIA in 1 patient 25
should be INVOLVED .....
---NNT=25 is not clinical significant.....hence choice B......
about the other choices....
Choice D....once you find the difference is STATISTICALLY SIGNIFICANT
...the sample size should be assumed LARGE and Enough...Only question
sample size if the STUDY is not STATISTICALLY SIGNIFICANT....i.e When
Calculated P Value is greater than .05.....
Choice C....Many times the investigator will report the lowest COMPUTED P
Value ....In general the smaller the p the BETTER......
And I remember a question on NBME 7 where the computed P value was
greater than .05.....and the answer was ....Due to chance alone....but the
scenario was almost the same as this question.....

5.BB
HSV-1 is the more common cause of adult encephalitis. HSV-2 is the more
common cause of newborn encephalitis. The encephalitis affects the
temporal lobes of the brain in most cases
6.BB
Pancreatic cancer is sometimes called a "silent killer" because early
pancreatic cancer often does not cause symptoms,[4] and the later
symptoms are usually nonspecific and varied.[4] Therefore, pancreatic
cancer is often not diagnosed until it is advanced.
Painless jaundice (yellow tint to whites of eyes and/or yellowish skin in
serious cases, possibly in combination with darkened urine) when a cancer of
the head of the pancreas (about 60% of cases) obstructs the common bile
duct as it runs through the pancreas. This may also cause pale-colored stool
and steatorrhea. The jaundice may be associated with itching as the salt
from excess bile can cause skin irritation.
7-D Secretin is a hormone that controls the secretions into the
duodenum, and also separately, water homeostasis throughout the body. It
is produced in the S cells of the duodenum in the crypts of Lieberkhn.[1]
Its effect is to regulate the pH of the duodenal contents via the control of
gastric acid secretion and buffering with bicarbonate from the centroacinar
cells of the pancreas as well as intercalated ducts. It is notable for being the
first hormone to be identified. In humans, the secretin peptide is encoded by
the SCT
8-E Opioids withdrawal produce, anxiety, insomnia, anorexia, sweating
dilated pupils, piloerection (cold turkey), fever, rhinorrhea, nausea, stomach
cramps, diarrhea (flulike symtoms) and yawning
9-D Raynaud's phenomenon is an exaggeration of vasomotor
responses to cold or emotional stress. More specifically, it is a
hyperactivation of the sympathetic system causing extreme vasoconstriction
of the peripheral blood vessels, leading to tissue hypoxia
Nefedipine vasodilate by releasing nitric oxide in smooth muscle, causing
increased in cGMP and smooth muscle relaxation,. dilate veins more than
arteries and dicreased the preload

10.C
An exudate is any fluid that filters from the circulatory system into lesions or
areas of inflammation.
Its composition varies but generally includes water and the dissolved solutes
of the main circulatory fluid such as sap or blood. In the case of blood: it will
contain some or all plasma proteins, white blood cells, platelets and (in the
case of local vascular damage) red blood cells.
Transudate Exudate
Main causes Increased hydrostatic
pressure,
Decreased colloid
osmotic pressure Inflammation
Appearance Clear Cloudy
Specific gravity < 1.012 > 1.020
Protein content < 2 g/dL > 2.9 g/dL
fluid protein
serum protein < 0.5 > 0.5
Difference of
albumin content
with blood albumin > 1.2 g/dL < 1.2 g/dL
fluid LDH
upper limit for serum < 0.6 or < > 0.6[5] or >
Cholesterol content < 45 mg/dL > 45 mg/dL[5

Exudate...An exudate is any fluid that filters from the circulatory


system into lesions or areas of inflammation.
it will contain some or all plasma proteins, white blood cells, platelets and (in
the case of local vascular damage) red blood cells
protein rich....specific gravity >1020
***Types***
Purulent or suppurative exudate consists of plasma with both active and
dead neutrophils, fibrinogen, and necrotic parenchymal cells. This kind of
exudate is consistent with more severe infections, and is commonly referred
to as pus.
Fibrinous exudate is composed mainly of fibrinogen and fibrin. It is
characteristic of rheumatic carditis, but is seen in all severe injuries such as
strep throat and bacterial pneumonia. Fibrinous inflammation is often
difficult to resolve due to the fact that blood vessels grow into the exudate
and fill the space that was occupied by fibrin. Often, large amounts of
antibiotics are necessary for resolution.
Catarrhal exudate is seen in the nose and throat and is characterized by a
high content of mucus.
Serous exudate (sometimes classified as serous transudate) is usually seen
in mild inflammation, with little protein content. Its consistency resembles
that of serum, and can usually be seen in certain disease states like
tuberculosis. (See below for difference between transudate and exudate)
Malignant (or cancerous) pleural effusion is effusion where cancer cells are
present. It is usually classified as exudate.
Trasudate versus Exudate
Hypocellular,
Protein poor
Specific gravity
11.DD
Rigt heart failure....Jugular venous distantion and bilateral Ankle,sacral
edema ..due to inc venous pressure

12.FF
The dissociation curve shifts to the right when carbon dioxide or hydrogen
ion concentration is increased. This facilitates increased oxygen dumping.
This mechanism allows for the body to adapt the problem of supplying more
oxygen to tissues that need it the most.
The general equation for the Haldane Effect is: H+ + HbO2 H+Hb + O2
.It has asked the effect of shift of the O2 dissociation curve to the right due
to an increase of H+...
The answer should definitely be F,i.e., release of oxygen in tissue capillaries.
That's what happens when pH falls and the curve shifts to the right.The
unloading of O2 becomes easier in tissue capillaries.They have not
mentioned anything regarding hypoxia in the question which u mentioned
here.
Rise in 2,3 BPG will also have the same effect, but the rise in 2,3 DPG is not
solely due to rise in H+...
2,3-Bisphosphoglyceric acid ....interacts with deoxygenated hemoglobin beta
subunits by decreasing their affinity for oxygen, so it allosterically promotes
the release of the remaining oxygen molecules bound to the hemoglobin,
thus enhancing the ability of RBCs to release oxygen near tissues that need
it most.

13.CC

A fat embolism is a type of embolism that is often (but not always)


caused by physical trauma like fracture of long bones, soft tissue trauma and
burns.
Fat emboli occur in almost 90% of all patients with severe injuries to bones,
although only 10% of these are symptomatic. The risk of fat embolism
syndrome is thought to be reduced by early immobilization of fractures and
especially by early operative correction. There is also some evidence that
steroid prophylaxis of high-risk patients reduces the incidence. The mortality
rate of fat-embolism syndrome is approximately 10-20%.
The pathogenesis occurs due to both mechanical obstruction and biochemical
injury. The microemboli cause pulmonary and cerebral microvasculature
occlusion. It is aggravated by local platelet and erythrocyte aggregation. The
release of free fatty acids from the fat globules causes local toxic injury to
endothelium. The vascular damage is aggravated by platelet activation and
recruitment of granulocytes.
http://library.med.utah.edu/WebPath/HISTHTML/STAINS/STAIN011.html
14.AA

http://www.google.com/imgres?
imgurl=http://missinglink.ucsf.edu/lm/ids_104_ce...0CEAQ9QEwAw

15.BB
Coccidioidmycosis....mold in soil.....Southwestern US,California...causes
pneumonia and meningitis(very rare).
Can disseminate to bone and skin
Culture...25degree...Hyphe with doubly reflective wall
Biopsy Thick -wall spherules filled with endospores
16.AA
In statistics, analysis of variance (ANOVA) is a collection of statistical
models, and their associated procedures, in which the observed variance in a
particular variable is partitioned into components attributable to different
sources of variation. In its simplest form ANOVA provides a statistical test of
whether or not the means of several groups are all equal, and therefore
generalizes t-test to more than two groups. ANOVAs are helpful because
they possess an advantage over a two-sample t-test. Doing multiple twosample t-tests would result in an increased chance of committing a type I
error. For this reason, ANOVAs are useful in comparing two, three or more
means.

1-"Interval variable" (values from where we can drive their "Mean")


e.g. measurements of B.P, Weight, Cholesterol levels etc
2-"Nominal variable" ( different categories or "GROUPS")
e.g.
*Gender ---is a Nominal that has Two Groups (Male and Female)
*AGE-------is a Nominal that we can have either TWO groups (Elderly and
Middle age) or
THREE Groups(Elderly, Middle-age and Children) etc
*Race------a Nominal that can have many groups
Two groups (African Americans and Latino)
Three groups(African Americans, Latino and Asian) and so forth
e.g.
*"GENDER"(Men and Women)....................................................(Single
Nominal)
*"GENDER"(Men and Women) n "RACE"(Caucasian and African)....(Two
Nominals )

e.g we compare Blood Pressure measurements


*INTERVAL variable ("MEANS" of B.P measurements)
*NOMINAL variable ("GROUPS" )
so to compare MEANS of B.P. b/w two are more groups we do these tests
t-Test:
1 Interval (Blood pressure Means)
1 Nominal with "ONLY TWO GROUPS"
e.g. comparing the BP means b/w (Latinos) and (African Americans)
*for a t-Test it will be a wrong statement to say
e.g comparing B.P means b/w (Latinos), (African Americans) and
(Asians)....*got 3 groups
Now what's the difference B/W t-Test and ANOVA

t-Test------>ONLY TWO Groups comparison


ANOVA------------>TWO Groups or more than Two Groups comparison
NOTE:these two will give u the Identical results if comparing "TWO Groups"
t-Test................... 1 Interval and 1 Nominal with TWO Groups
One-way ANOVA.....1 Interval and 1 Nominal with TWO Groups
and for the above reason they rather use One-way ANOVA when more than
two groups are there to be compared e.g as above
comparing B.P means b/w (Latinos), (African Americans) and (Asians).......3
groups
Two-way ANOVA:
1 Interval
2 Nominal e.g. Gender (Men and Women) *AND Race (Caucasians and
African Americans)
---->we do these tests to see If there is a Difference b/w groups ?
t-statistics (t-Test) and f-statistics(ANOVA) will be compared to get a p-value
If p-value = 0.05 or less -----it is said to be "Statistically Significant' and we
say that there is a Difference in these Groups.

Chi-square:
we compare b/w "TWO NOMINAL" (with ANY # of GROUPS)
(Note; we are not using any Interval variable here , just the NOMINAL data)
best example...."Testing Drug Efficacy"
First Nominal........(New drug group) and (Placebo group)
Second Nominal ...(Recovered group) and(Not recovered group)
***I think this is all u have to do is recognize what kind of data is
presented , how many Nominals, How many groups and If Interval variable
is used or not?
1 Interval and 1 Nominal (Two Groups).......................t-Test or One-way

ANOVA
1 Interval and 1 Nominal (More than Two groups)........One-way ANOVA
1 Interval and *2 Nominal (many groups)....................Two-way ANOVA
.....................*2 Nominal (any# of Groups).................Chi-square
17.DD
Post-infectiousPost-infectious glomerulonephritis can occur after essentially
any infection, but classically occurs after infection with Streptococcus
pyogenes. It typically occurs 1014 days after a skin or pharyngeal infection
with this bacterium.
Patients present with signs and symptoms of glomerulonephritis. Diagnosis
is made based on these findings in an individual with a history of recent
streptococcal infection. Streptococcal titers in the blood (antistreptolysin O
titers) may support the diagnosis
causes Nephritic syndrome...LM..glumeruli enlarged and hypercelullar
Neutrophiles..Lumpy -Bumpy appereance
Em...Subepithelial IC humps..IgM,IgG,C3
IF...granular
resolve spontaneously

18.EE
The thymus, is the principal organ responsible for the T cell's maturation.
All T cells originate from haematopoietic stem cells in the bone marrow.
Haematopoietic progenitors derived from haematopoietic stem cells populate
the thymus and expand by cell division to generate a large population of
immature thymocytes. The earliest thymocytes express neither CD4 nor
CD8, and are therefore classed as double-negative (CD4-CD8-) cells. As they
progress through their development they become double-positive
thymocytes (CD4+CD8+), and finally mature to single-positive (CD4+CD8or CD4-CD8+) thymocytes that are then released from the thymus to
peripheral tissues.
About 98% of thymocytes die during the development processes in the
thymus by failing either positive selection or negative selection, whereas the
other 2% survive and leave the thymus to become mature
immunocompetent T cells.
Positive selection "selects for" T-cells capable of interacting with MHC.
Negative selection removes thymocytes that are capable of strongly binding
with "self" peptides presented by MHC.

19.
B: Interfere with Carboxylation of Coagulation Factors.

The answer should be B as they have asked which drug is to be given


for 6 months after initiating therapy with heparin.
Typically, before initiating warfarin therapy we start heparin to obtain
immediate anticoagulation as warfarin takes time for it action.
Immadiate anticoagulation for pulmonary embolism,stroke ,acute coronary
syndrome
Toxicity...bleeding Thrombocytopenis...HIT,osteoparosis,drug-drug
interactions
For rapid reversalof heparinization....PROTAMINE SULFATE
****Newer low molecular weight heparin...Enoxaparin act more on Xa
have better bioavilibility without laboratory monitoring
20-E "Drug Y alone no effect" ( partial agonist are incapable of eliciting
a maximal response and are less efecctive than the full agonist

D-Noncompetitive Antagonist
This is the case on the concept of "Spare Receptors"
Assume ...100% receptor occupancy required by an Agonist to exert
Maximum Effect (Emax) /Optimal effect.
and now consider, if Emax is achieved with result
*Efficacy of Drug X decreases
*5 of the spare receptors are still Unoccupied
Now if we raise the drug "X" dosage.....it will bind to the remaining
unoccupied spare receptors and brings it Efficacy back to to the Emax.
there for "No Change in Efficacy" , only "Potency decreases"(as needed
higher dose of Drug X)
---------------------------------Graph on the Right-------------------------But at "High Dose" of of NCA (Drug Y), it will bind to e.g all the 10 spare
receptors---->result
*Efficacy(Emax) of Drug X decreases
*there are non of the spare receptors left unoccupied
since there are no extra receptors left, even if we raise the Drug "X" dosage,
it won't be able to increase it's Efficacy back to Emax.
21-E In treating hepatic encephalopathy, lactulose helps "draw out"
ammonia (NH3) from the body.[5]
Lactulose is metabolized in the colon by bacterial flora to short chain fatty
acids including the production of the lactic acid and acetic acid. This partially
dissociates, acidifying the colonic contents (increasing the H+ concentration
in the gut).[6] This favors the formation of the nonabsorbable NH4+ from
NH3, trapping NH3 in the colon and effectively reducing plasma NH3
concentrations.
The effectiveness of lactulose in treating hepatic encephalopathy is
somewhat controversial.[
Lactulose for hepatic encephalopathy generally requires oral dosage three or
four times a day with diarrhea almost a certain side effect.

22-E A pheochromocytoma or phaeochromocytoma (PCC) is a


neuroendocrine tumor of the medulla of the adrenal glands (originating in
the chromaffin cells), or extra-adrenal chromaffin tissue that failed to
involute after birth [1] and secretes excessive amounts of catecholamines,
usually noradrenaline (norepinephrine), and adrenaline (epinephrine) to a
lesser extent
TO ADD; The signs and symptoms of a pheochromocytoma are those of
sympathetic nervous system hyperactivity, including:
Skin sensations
Flank pain
Elevated heart rate
Elevated blood pressure, including paroxysmal (sporadic, episodic) high
blood pressure, which sometimes can be more difficult to detect; another
clue to the presence of pheochromocytoma is orthostatic hypotension (a fall
in systolic blood pressure greater than 20 mmHg or a fall in diastolic blood
pressure greater than 10 mmHg on making the patient stand)
Palpitations
Anxiety often resembling that of a panic attack
Diaphoresis (excessive sweating)
Headaches
Pallor
Weight loss
Localized amyloid deposits found microscopically
Elevated blood glucose level (due primarily to catecholamine stimulation of
lipolysis (breakdown of stored fat) leading to high levels of free fatty acids
and the subsequent inhibition of glucose uptake by muscle cells. Further,
stimulation of beta-adrenergic receptors leads to glycogenolysis and
gluconeogenesis and thus elevation of blood glucose levels).
A pheochromocytoma can also cause resistant arterial hypertension. A
pheochromocytoma can be fatal if it causes malignant hypertension, or
severely high blood pressure. This hypertension is not well controlled with
standard blood pressure medications

23.DD

we need to cut 2lb=7000 cal (3500*2)


1 hr walk/d will cut 3500 cal
In the meanwhile she is also getting 1800*7 calories=12600 in a week
decrease intake for 500 cal /d * 7 days=3500 cal
so 3500+3500=7000
It is a mathematic calculation...
She is asked to loss 2 pounds weight weekly...each pounds=3500 cal and
brisk walking consums 500cal per hour
so........................................................... 2 pounds=7000cal
she needs to loss 1000 cal per day......if she walked 2 hours per day....she
could lose 1000 calories
but between choices there is no 2 hours.so if she decreases 500 cal in her
diet and gets 1 hour brisk walking then she will loss 1000 cal/day.......and
7000/week

24. C
Systemic lupus erythematosus often abbreviated to SLE or lupus, is a
systemic autoimmune disease (or autoimmune connective tissue disease)
that can affect any part of the body. As occurs in other autoimmune
diseases, the immune system attacks the body's cells and tissue, resulting in
inflammation and tissue damage.[1] It is a Type III hypersensitivity reaction
caused by antibody-immune complex formation.
SLE most often harms the heart, joints, skin, lungs, blood vessels, liver,
kidneys, and nervous system. The course of the disease is unpredictable,
with periods of illness (called flares) alternating with remissions. In SLE, the
body's immune system produces antibodies against itself, particularly
against proteins in the cell nucleus. SLE is triggered by environmental
factors that are unknown. "All the key components of the immune system
are involved in the underlying mechanisms [of SLE]" according to Rahman,
and SLE is the prototypical autoimmune disease. The immune system must
have a balance (homeostasis) between being sensitive enough to protect
against infection, and being too sensitive and attacking the body's own
proteins (autoimmunity). From an evolutionary perspective, according to
Crow, the population must have enough genetic diversity to protect itself
against a wide range of possible infection; some genetic combinations result
in autoimmunity. The likely environmental triggers include ultraviolet light,
drugs, and viruses. These stimuli cause the destruction of cells and expose
their DNA, histones, and other proteins, particularly parts of the cell nucleus.
Because of genetic variations in different components of the immune
system, in some people the immune system attacks these nuclear-related
proteins and produces antibodies against them. In the end, these antibody
complexes damage blood vessels in critical areas of the body, such as the
glomeruli of the kidney; these antibody attacks are the cause of SLE.
Researchers are now identifying the individual genes, the proteins they
produce, and their role in the immune system. Each protein is a link on the
autoimmune chain, and researchers are trying to find drugs to break each of
those links.

Renal
Painless hematuria or proteinuria may often be the only presenting renal
symptom. Acute or chronic renal impairment may develop with lupus
nephritis, leading to acute or end-stage renal failure. Because of early
recognition and management of SLE, end-stage renal failure occurs in less
than 5% of cases.
A histological hallmark of SLE is membranous glomerulonephritis with "wire
loop" abnormalities.[16] This finding is due to immune complex deposition
along the glomerular basement membrane, leading to a typical granular
appearance in immunofluorescence testing.
Lupus nephritis is an inflammation of the kidney caused by systemic lupus
erythematosus (SLE), a disease of the immune system. Apart from the
kidneys, SLE can also damage the skin, joints, nervous system and virtually
any organ or system in the body.
Histologically a wire-loop lesion will be present. The wire loop lesion is a
glomerular capillary loop with subendothelial immune complex deposition
that is circumferential around the loop.
25. E
Kwashiorkor (pronounced /kw irkr/) is an acute form of childhood
protein-energy malnutrition characterized by edema, irritability, anorexia,
ulcerating dermatoses, and an enlarged liver with fatty infiltrates. The
presence of edema caused by poor nutrition defines kwashiorkor.[1]
Kwashiorkor was thought to be caused by insufficient protein consumption
but with sufficient calorie intake, distinguishing it from marasmus. More
recently, micronutrient and antioxidant deficiencies have come to be
recognized as contributory. Cases in the developed world are rare.
The defining sign of kwashiorkor in a malnourished child is pedal edema
(swelling of the feet). Other signs include a distended abdomen, an enlarged
liver with fatty infiltrates, thinning hair, loss of teeth, skin depigmentation
and dermatitis. Children with kwashiorkor often develop irritability and
anorexia.

Marasmus is a form of severe protein-energy malnutrition


characterized by energy deficiency.
A child with marasmus looks emaciated. Body weight may be reduced to less
than 80% of the average weight that corresponds to the height .[citation
needed] Marasmus occurrence increases prior to age 1, whereas kwashiorkor
occurrence increases after 18 months. It can be distinguished from
kwashiorkor in that kwashiorkor is protein wasting with the presence of
edema.
The malnutrition associated with marasmus leads to extensive tissue and
muscle wasting, as well as variable edema. Other common characteristics
include dry skin, loose skin folds hanging over the glutei, axillae, etc. There
is also drastic loss of adipose tissue from normal areas of fat deposits like
buttocks and thighs. The afflicted are often fretful, irritable, and voraciously
hungry. The word marasmus comes from a Greek word meaning
starvation. Marasmus is generally known as the gradual wasting away of the
body due to severe malnutrition or inadequate absorption of food. Marasmus
is a form of severe protein deficiency and is one of the forms of proteinenergy malfunction (PEM). It is a severe form of malnutrition caused by
inadequate intake of proteins and calories.
26. C
Primary polydipsia or psychogenic polydipsia is a special form of polydipsia.
[1] It is usually associated with a patient's increasing fluid intake due to the
sensation of having a dry mouth.
When the term "psychogenic polydipsia" is used, it implies that the condition
is caused by mental disorders. However, the dry mouth is often due to
phenothiazine medications used in some mental disorders, rather than the
underlying condition
The patient drinks large amounts of water, which dilutes the extracellular
fluid, decreasing its osmotic pressure. The body responds to this by
decreasing the level of vasopressin (antidiuretic hormone), with a resultant
increased production of urine (polyuria). This urine will have a low
electrolyte concentration.
Clinical presentation
Patients have been known to seek fluids from any source possible.
In extreme episodes, the patient's kidneys will be unable to deal with the
fluid overload, and weight gain will be noted.
Primary polydipsia can be life threatening as serum sodium is diluted to an
extent that seizures and cardiac arrest can occur.

27. A
Kawasaki disease: Acute, self-limiting necrotizing vasculitis in infants/
children. Association with Asia ethnicity. Fever, conjuntivitis, changes in lips/
oral mucosa (strawberry tongue), lymphadenitis, desquamative skin rash.
May develop coronary aneurysms.
28. D
In genetic medicine, a mosaic or mosaicism denotes the presence of two
populations of cells with different genotypes in one individual who has
developed from a single fertilized egg.[1] Mosaicism may result from a
mutation during development which is propagated to only a subset of the
adult cells.
Somatic mosaicism
Somatic mosaicism occurs when the somatic cells of the body are of more
than one genotype. In the more common mosaics, different genotypes arise
from only a single fertilized egg cell, due to mitotic errors at first cleavage.
Another form of somatic mosaicism is chimerism, where two or more
genotypes arise from the fusion of more than one fertilized zygote in the
early stages of embryonal development.
In rare cases, intersex conditions can be caused by mosaicism where some
cells in the body have XX and others XY chromosomes.[2][3]
The most common form of mosaicism found through prenatal diagnosis
involves trisomies. Although most forms of trisomy are due to problems in
meiosis and affect all cells of the organism, there are cases where the
trisomy occurs in only a selection of the cells. This may be caused by a
nondisjunction event in an early mitosis, resulting in a loss of a chromosome
from some trisomic cells.[4] Generally this leads to a milder phenotype than
in non-mosaic patients with the same disorder.
An example of this is one of the milder forms of Klinefelter's syndrome,
called 46/47 XY/XXY mosaic wherein some of the patient's cells contain XY
chromosomes, and some contain XXY chromosomes. The 46/47 annotation
indicates that the XY cells have the normal number of 46 total
chromosomes, and the XXY cells have 47 total chromosomes.
Around 30% of Turner's syndrome cases demonstrate mosaicism, while
complete monosomy (45 XO) occurs in about 5060% of cases.
True mosaicism should not be mistaken for the phenomenon of Xinactivation, where all cells in an organism have the same genotype, but a
different copy of the X chromosome is expressed in different cells, such as in
calico cats.

29.C
PaCO2=metabolic CO2 production/Va
So if Ventilation increases to match up with CO2 production, PCO2 remains
the same
30-F The classic triad for congenital rubella syndrome is:
Sensorineural deafness (58% of patients)
Eye abnormalitiesespecially cataract and microphthalmia (43% of patients)
Congenital heart diseaseespecially patent ductus arteriosus (50% of
patients)
Other manifestations of CRS may include:
Spleen, liver or bone marrow problems (some of which may disappear
shortly after birth)
Mental retardation
Small head size (microcephaly)
Eye defects
Low birth weight
Thrombocytopenic purpura (presents as a characteristic blueberry muffin
rash)
Hepatomegaly
Micrognathia
Children who have been exposed to rubella in the womb should also be
watched closely as they age for any indication of the following:
Developmental delay
Autism spectrum disorders[1]
Schizophrenia[2]
Growth retardation
Learning disabilities
Diabetes
Glaucoma
Note:NO a congenital cytomegalovirus Generalized infection may occur in
the infant, and can cause complications such as low birth weight,
microcephaly, seizures, petechial rash similar to the "blueberry muffin" rash
of congenital rubella syndrome, and moderate hepatosplenomegaly (with
jaundice).

31. C
Pityriasis or tia versicolor caused by malassezia furfur. Degradation of lipids
produces acids that damage melanocytes and cause hypopigmented patches.
Occurs in hot humid weather.
Yeast clusters & and short curved septate hyphae KOH scaping shows
spaghetti and meatballs.
Malassezia (formerly known as Pityrosporum) is a genus of fungi. Malassezia
is naturally found on the skin surfaces of many animals, including humans.
In occasional opportunistic infections, some species can cause
hypopigmentation on the trunk and other locations in humans.
T: Topical Miconazole, selenium sulfide.
32. A
N. meningitidis:
Gram neg. kidney bean shaped diploccoci, large capsule; latex particule
agluttination ( or Counter immunoelectrophoresis ( CIE) to identify N.
meningitidis capsular antigen in CSF.
Reservorio: Human nasopharynx
Transmision: Respiratory droplets; oropharyngeal colonization, spread to
meninges via blood stream.
Pathogenesis: Important virulent factor:
Polysacharide capsule: antiphagocytic, antigenic, 5 common serogrups : B is
not strongly immunogenic( sialid acid), B strain is most common strain in
US.
(Kaplan Microbiology book page 257)

33. D Leuprolide
As a medical term, precocious puberty describes puberty occurring at an
unusually early age. In most of these children, the process is normal in
every respect except the unusually early age, and simply represents a
variation of normal development. In a minority of children, the early
development is triggered by a disease such as a tumor or injury of the brain.
Even in instances where there is no disease, unusually early puberty can
have adverse effects on social behavior and psychological development, can
reduce adult height potential, and may shift some life-long health risks.
Central precocious puberty can be treated by suppressing the pituitary
hormones that induce sex steroid production.
The term is used with several slightly different meanings that are usually
apparent from the context. In its broadest sense, and often simplified as
early puberty, "precocious puberty" sometimes refers to any physical sex
hormone effect, due to any cause, occurring earlier than the usual age,
especially when it is being considered as a medical problem. Stricter
definitions of "precocity" may refer only to central puberty starting before a
statistically specified age based on percentile in the population (e.g., 2.5
standard deviations below the population mean),[1] on expert
recommendations of ages at which there is more than a negligible chance of
discovering an abnormal cause, or based on opinion as to the age at which
early puberty may have adverse effects. A common definition for medical
purposes is onset before 8 years in girls or 9 years in boys.
One possible treatment is with anastrozole. Histrelin acetate (Supprelin LA),
Triptorelin or Leuprolide, any GnRH agonists, may also be used. GnRH
agonists stimulate the pituitary to release Follicle Stimulating Hormone
(FSH) and Luteinizing Hormone (LH). However, when used regularly, GnRH
agonists cause a decreased release of FSH and LH. GnRH produced by the
hypothalamus is pulsatile, allowing for a physiologic release of FSH and LH.

34-B Inhibitory synapses Postsynaptic CA2+ Influx


The neurotransmitter at inhibitory synapses hyperpolarizes the postsynaptic
membrane.
Example: gamma aminobutyric acid (GABA) at certain synapses in the brain.
The GABAA receptor is a ligand-gated chloride channel. Binding of GABA to
the receptors increases the influx of chloride (Cl) ions into the postsynaptic
cell raising its membrane potential and thus inhibiting it.
This is a fast response taking only about 1 millisecond.
Binding of GABA to GABAB receptors activates an internal G protein and a
"second messenger" that leads to the opening of nearby potassium (K+)
channels. As you might expect, this is a slower response, taking as long as 1
second.
In both cases, the resulting facilitated diffusion of ions (chloride IN;
potassium OUT) increases the membrane potential (to as much as 80 mv).
This increased membrane potential is called an inhibitory postsynaptic
potential (IPSP) because it counteracts any excitatory signals that may
arrive at that neuron.
A hyperpolarized neuron appears to have an increased threshold. Actually,
the threshold voltage (about 50 mv) has not changed. It is simply a
question of whether the depolarization produced by excitatory synapses on
the cell minus the hyperpolarizing effect of inhibitory synapses can reach this
value or not.
Mutation of the GABA, (no receptor) secundary to the mutation DECREASED
THE influx of Cl-

35-F (DCT!) ADH is secreted in response to increased plasma


osmolarity and dicreased blood volume, binds to receptors on principal cells,
causing increased number of water channels and increased water
reabsortion in the collecting tube. FA pag 396
The main effector organ for fluid homeostasis is the kidney. ADH acts by
increasing water permeability in the collecting ducts and distal convoluted
tubules, specifically it acts on proteins called aquaporins which open to allow
water into the collecting duct cells. This increase in permeability allows for
reabsorption of water into the bloodstream, thus concentrating the urine.
Diabetes insipidus (DI) is a condition characterized by excessive thirst and
excretion of large amounts of severely diluted urine, with reduction of fluid
intake having no effect on the latter. There are several different types of DI,
each with a different cause. The most common type in humans is central DI,
caused by a deficiency of arginine vasopressin (AVP), also known as
antidiuretic hormone (ADH). The second common type of DI is nephrogenic
diabetes insipidus, which is caused by an insensitivity of the kidneys to ADH.
It can also be an iatrogenic artifact of drug use.

36.CC
Subdural hematoma....rupture of bridging vein
crescent -shaped hemorrhage that crosses suture lines...cn not cross
falx.tentorium
venous bleeding with delayed onset ,seen in elderly,alcoholics,blunt
trauma,shaken baby
37 A esophageal varices
liver cirrhosis -->portal hypertension -->Ascites, splenomegaly, esophageal
varices, caput medusae, haemarrhoids
38 B Zona glomerulosa -->Aldosterone
other options C -zona fasciculata, D -zona reticularis, E -adrenal medulla
39.BB
Diabetic retinopathy is retinopathy (damage to the retina) caused by
complications of diabetes mellitus, which can eventually lead to blindness. It
is an ocular manifestation of systemic disease which affects up to 80% of all
patients who have had diabetes for 10 years or more.
Diabetic retinopathy is the result of microvascular retinal changes.
Hyperglycemia-induced intramural pericyte death and thickening of the
basement membrane lead to incompetence of the vascular walls. These
damages change the formation of the blood-retinal barrier and also make
the retinal blood vessels become more permeable.
Small blood vessels such as those in the eye are especially vulnerable to
poor blood sugar (blood glucose) control. An overaccumulation of glucose
and/or fructose damages the tiny blood vessels in the retina. During the
initial stage, called nonproliferative diabetic retinopathy (NPDR), most people
do not notice any change in their vision.
Some people develop a condition called macular edema. It occurs when the
damaged blood vessels leak fluid and lipids onto the macula, the part of the
retina that lets us see detail. The fluid makes the macula swell, which blurs
vision.
40 B chemokine receptor
HIV entry requires interaction with CD4 receptors and CCR5 receptors
41 C
its not A for sure

42.AA
Adenocarcinoma is a type of cancer that develops in cells lining glandular
types of internal organs,The World Health Organization (WHO) defines
adenocarcinoma as "a malignant epithelial tumor with tubular, acinar, or
papillary growth patterns, and/or mucus production by the tumor
cells."Currently the WHO recognizes four categories of adenocarcinoma:
acinar
papillary
bronchioloalveolar
solid carcinoma with mucus formation
The majority of adenocarcinomas occur at the periphery of the lung, and, as
a resultare often asymptomatic until late in their course. They frequently lie
just below the pleura, and cause pleural retraction and thickening on x-ray.
Often adenocarcinomas are discovered on routine chest x-rays or in a
primary search for distant metastases.
Adenocarcinomas grossly present with the "three P's" - peripheral,
pigmented and puckered. Commonly lesions are found near the pleural
surface (peripheral) which is retracted (puckered) over the neoplasm.The cut
surface is often white .
43.DD
Physiologically, phosphorylase kinase plays the important role of stimulating
glycogen breakdown into free glucose by phosphorylating glycogen
phosphorylase and stabilizing its active conformation. This activity is
particularly important in liver and muscle cells, though for somewhat
different purposes. While muscle cells generally break down glycogen to
power their immediate activity, liver cells are responsible for maintaining
glucose concentration in the bloodstream. Thus, the regulatory mechanisms
of PhK activity vary somewhat depending on cell type.
44 A oocyte donation
turners sundrome --- streak ovaries-- no germ cells
bosted by nf2011
to add
Genetic mosaicism (46XX/45XO) is most often implicated, alongside
nondisjunction (45XO) and partial monosomy (46XX).

Turner syndrome is characterized by primary amenorrhea, premature


ovarian failure, streak gonads and infertility. However, technology (especially
oocyte donation) provides the opportunity of pregnancy in these patients.
45 B inheritance of amplified CYP2D6 locus
rule out A induction --->increased dose should induce a response
C inactive alleles & D inhibitor --> small doses should do the trick
E tolerance ----> initiall doses should induce a response
45 B inheritance of amplified CYP2D6 locus
rule out A induction --->increased dose should induce a response
C inactive alleles & D inhibitor --> small doses should do the trick
E tolerance ----> initiall doses should induce a response
*************************************************************
********************************
46.EE
Ureters paa under uterine artery and under ductus deferens.
Complication in surgery...ligation the ovarian vasculature in the suspensory
lig.....accidental ligationof the Ureter
47.EE
Microscopic Polyangitis...like wegner but lacks granuloma....P-ANCA positive
All affect small vessels

48.DD
Weber's syndrome (superior alternating hemiplegia) is a form of stroke
characterized by the presence of an oculomotor nerve palsy and
contralateral hemiparesis or hemiplegia.
This lesion is usually unilateral and affects several structures in the midbrain
including:
substantia nigra contralateral parkinsonism because its dopaminergic
projections to the basal ganglia innervate the ipsilateral hemisphere motor
field, leading to a movement disorder of the contralateral body.
corticospinal fibers contralateral hemiparesis and typical upper motor neuron
findings
corticobulbar tract difficulty with contralateral lower facial muscles and
hypoglossal nerve functions
oculomotor nerve fibers ipsilateral oculomotor nerve palsy with a drooping
eyelid and fixed wide pupil pointed down and out. This leads to diplopia
It is caused by midbrain infarction as a result of occlusion of the paramedian
branches of the posterior cerebral artery or of basilar bifurcation perforating
arteries

Thalamic syndrome (or thalamic pain syndrome)It is also known as


"Dejerine-Roussy disease"
is a condition that can be associated with inadequate blood supply from the
posterior cerebral artery. It is a rare neurological disorder in which the body
becomes hypersensitive to pain as a result of damage to the thalamus, a
part of the brain that affects sensation. The thalamus has been described as
the brains sensory relay station. Primary symptoms include pain and loss of
sensation, usually in the face, arms, and/or legs.
Pain or discomfort may be fel after being mildly touched or even in the
absence of a stimulus. The pain associated with thalamic syndrome may be
made worse by exposure to heat or cold and by emotional distress.
Sometimes, this may include even such emotions as those brought on by
listening to music.
Gerstmann's syndrome
is a cognitive impairment that results from damage to a specific area of the
brain -- the left parietal lobe in the region of the angular gyrus. It may occur
after a stroke or in association with damage to the parietal lobe. It is
characterized by four primary symptoms: a writing disability (agraphia or
dysgraphia), a lack of understanding of the rules for calculation or arithmetic
(acalculia or dyscalculia), an inability to distinguish right from left, and an
inability to identify fingers (finger agnosia). The disorder should not be
confused with Gerstmann-Strussler-Scheinker disease, a type of
transmissible spongiform encephalopathy.

Lateral medullary syndrome (also called Wallenberg syndrome and posterior


inferior cerebellar artery syndrome) is a disease in which the patient has a
constellation of neurologic symptoms due to injury to the lateral part of the
medulla in the brain, resulting in tissue ischemia and necrosis.
This syndrome is characterized by sensory deficits affecting the trunk (torso)
and extremities on the opposite side of the infarction and sensory deficits
affecting the face and cranial nerves on the same side with the infarct.
Specifically, there is a loss of pain and temperature sensation on the
contralateral (opposite) side of the body and ipsilateral (same) side of the
face. This crossed finding is diagnostic for the syndrome.
Clinical symptoms include swallowing difficulty, or dysphagia, slurred speech,

ataxia, facial pain, vertigo, nystagmus, Horner syndrome, diplopia, and


possibly palatal myoclonus.

49 E zolpidem
Zolpidem (Ambien) is a prescription medication used for the short-term
treatment of insomnia, as well as some brain disorders. It is a short-acting
nonbenzodiazepine hypnotic that potentiates gamma-aminobutyric acid
(GABA), an inhibitory neurotransmitter, by binding to GABAA receptors at
the same location as benzodiazepines. It works quickly (usually within 15
minutes) and has a short half-life (23 hours).
Zolpidem has not adequately demonstrated effectiveness in maintaining
sleep, however it is effective in initiating sleep

50.BB
Fragile X syndrome is a genetic disorder caused by mutation of the FMR1
gene on the X-chromosome.
Expansion of the CGG repeating codon to such a degree results in a
methylation of that portion of the DNA, effectively silencing the expression of
the FMR1 protein.
Mutation of the FMR1 gene leads to the transcriptional silencing of the fragile
X-mental retardation protein, FMRP. In normal individuals, FMRP is believed
to regulate a substantial population of mRNA: FMRP plays important roles in
learning and memory,

Block 3
1.EE
Sucralfate and bismuth....bind to ulser base....pysical protection,and allow
bicarbonate secretion to restablish PH gradient in the mucous layer
Misoprostol....a PGE1 analog.....inc.production and secretion of gastric
mucous barrier and dic acid production
A,B..can affect absorbtion,bioavailibility or urinary excreation of other drugs
by altering gastric and urinary PHor by delaying gastric emptying
Ranitidin....H2 blocker...reversible block of histamine H2 receptors.....dec H+
secretion by parietal cells
2.AA
CFTR gene on chromosome 7 ..deletion of Phe 508,folding of the CFTR
protein in the endoplasmic reticulum is affected by this mutation

3.BB
Digoxin toxicity......inc.parasympathetic
activity..nausea,vomitingdiarrhea,blurry yellow visionand arrhythmia
may cause inc.PR and decQT,scooping of STad T-wave inversion
Antidote...slowly normalize K+and Mg+,lidocain,anti-dig fragments...
4.CC
The concept they are testing is if a disease has a high heterozygote
frequency then the chances of a female carrier and a male carrier meeting
are higher, and thus chances of having a child with a recessive disorder in a
family where none of the grandparents had a disease are also higher.
It is referring to the particular gene frequency in the population. For
example, the gene frequency for cystic fibrosis is high in Caucasians.
Cystic Fibrosis....AR with high heterozigot frequency, most common lethal
genetic disease of caucasians.......deletion of Phe 508,folding of the CFTR
protein in the endoplasmic reticulum is affected by this mutation
defective Cl- chanel.......secretion abnormally thick mucous that pluge
lung ,pancreas and lver.....recurent pulmonary infection,Fat-soluble vitamine
deficiency....failure to thrive in infancy

5.CC
Spirochete Infections: Lyme Disease and Leptospirosis
Lyme disease is characterized by dermatological, neurological, cardiac,
rheumatic, and ophthalmic manifestations that result from tick-borne
transmission of the spirochete Borrelia burgdorferi
Three chronological stages:
1.Primary or initial phaserash at the site of tick bite (erythema chronicum
migrans) and flu-like symptoms.
2.Secondary or dissemination stagefurther dermatological, cardiac, and
neurological manifestations.
3.Tertiary or late stagearthritis, meningoencephalitis, cranial neuropathy,
peripheral neuropathy, carditis.
ASSOCIATED FEATURES
Conjunctivitis, most common manifestation in stage 1.
Cranial nerve palsies, optic nerve inflammation can occur in stage 2.
Corneal, uveal, and retinal inflammation can occur in stage 3.
Leptospirosis is a zoonotic infection of worldwide distribution caused by
pathogenic Leptospira species.
Biphasic disease.
Leptospiremic phasesevere headache, fever, myalgia.
Immune phasehigh fever, meningismus, central and peripheral nervous
system manifestations.
Ocular manifestations occur in the majority of patients.
Conjunctivitis, uveitis, subconjunctival hemorrhages.
ASSOCIATED FEATURE
Weils diseasesevere disease with hemorrhages, renal failure, and
jaundice.
6.DD
the bullet is seen inside the spleen

7.BB
The lesser omentum (small omentum; gastrohepatic omentum; omentum
minus) is the double layer of peritoneum that extends from the liver to the
lesser curvature of the stomach and the start of the duodenum.
The lesser omentum is extremely thin, and is continuous with the two layers
of peritoneum which cover respectively the antero-superior and posteroinferior surfaces of the stomach and first part of the duodenum.
http://en.wikipedia.org/wiki/File:Gray1104.png

8.CC
The paraaortic lymph node group is divided into three subgroups: preaortic,
retroaortic, and right and left lateral aortic.
The preaortic group drains the abdominal part of the gastrointestinal tract
above the mid-rectum.
The retroaortic group drains from the lateral and preaortic glands.
The lateral group drains the iliac lymph nodes, the ovaries, and other pelvic
organs. The lateral group nodes are located adjacent to the aorta, anterior to
the spine, extending laterally to the edge of the psoas major muscles, and
superiorly to the crura of the diaphragm.
9.EE
Otitis media is most commonly caused by infection with viral, bacterial, or
fungal pathogens. The most common bacterial pathogen is Streptococcus
pneumoniae.
Others include Pseudomonas aeruginosa, nontypeable Haemophilus
influenzae, and Moraxella catarrhalis.
Among older adolescents and young adults, the most common cause of ear
infections is Haemophilus influenzae. Viruses such as respiratory syncytial
virus (RSV) and those that cause the common cold may also result in otitis
media by damaging the normal defenses of the epithelial cells in the upper
respiratory tract.

10.AA
most human rabies cases in the United States were caused by bat bites that
probably were unrecognized or undetected,Rabis is a viral disease that
causes acute encephalitis .
The period between infection and the first flu-like symptoms is normally two
to twelve weeks, but can be as long as two years. Soon after, the symptoms
expand to slight or partial paralysis, cerebral dysfunction, anxiety, insomnia,
confusion, agitation, abnormal behavior, paranoia, terror, hallucinations,
progressing to delirium
The production of large quantities of saliva and tears coupled with an
inability to speak or swallow are typical during the later stages of the
disease; this can result in hydrophobia, in which the patient has difficulty
swallowing because the throat and jaw become slowly paralyzed, shows
panic when presented with liquids to drink, and cannot quench his or her
thirst.
Negri bodies are 100% diagnostic for rabies infection, but are found in only
about 80% of cases
11.DD
Viral vaccines that living strains elicit both-cell mediated and humoral
immunity, whereas killed viral vaccines elicit predominantly an antibody
respond....so common features of these two types... sabin and salk ...induce
humoral immunity/antibody respond..
12.CC
Albinism...Congenital deficiency of either
**
Tyrosinase ....inability to synthetize melanin from tyrosine...Autosomal
recessive
or
**
Defective tyrosine transporter...dec.amount of tyrosin and thus melanin
13.FF
....odds of having disease in exposed group divided by odds of having
disease in unexposed group
250/50/250/150......3

14.CC
Decreased ATP causes decreased action of Na+ / K+ pumps in the cell
membranes, leading to increased Na+ and water within the cell (cell
swelling).
15.FF
Individuals with passive-aggressive personality disorder appear to comply or
act appropriately, but actually behave negatively and passively resist. This
personality disorder is a chronic condition, meaning that it lasts throughout
life.
Symptoms include:
Contradictory and inconsistent behaviorAn individual with passiveaggressive personality disorder may appear enthusiastic to carry out others
requests, but he purposely performs in a manner that is not useful and
sometimes even damaging.
Intentional avoidance of responsibility. Some behaviors that may be used to
avoid responsibility include:
Procrastinationto delay or postpone needlessly and intentionally
Deliberate inefficiencypurposefully performing in an incompetent manner
Forgetfulness
Feelings of resentment toward others
Stubbornness
Argumentative, sulky, and hostile, especially toward authority figures
Easily offended
Resentful of useful suggestions from others
Blames others
Chronically impatient
Unexpressed anger or hostility
16.AA
supernumerary nipples (polythelia) more than the normal number of nipples,
may be on the breast or other parts of the body-found along the "milk line"
from axilla to groin. These accessory nipples resemble raised nevi
(commonly called "moles").
It is derived from epithelial tissue .
Usually, the supernumerary nipple remains undetected or asymptomatic.
Occasionally, the supernumerary nipple is noticed only when hormonal
changes during adolescence, menstruation, or pregnancy cause increased
pigmentation, fluctuating swelling, tenderness, or even lactation.

17.BB
In people who have sarcoidosis, immune system cells cause inflammation
and cluster to form lumps called granulomas.
If many granulomas form in an organ, they can affect how the organ works.
This can cause signs and symptoms. Signs and symptoms vary depending on
which organs are affected. Many people who have sarcoidosis have no
symptoms or mild symptoms.
Some researchers think that sarcoidosis develops when the immune system
responds to a trigger, such as bacteria, viruses, dust, or chemicals. Genetics
also may play a role in sarcoidosis.

18.CC
Influenza viruses...Orthomyxovirus...enveloped,single stranded..RNA viruses
with segmented genome.
Contain:
Hemagglutinin antigen....promote viral entery and
Neuroaminidase antigen..promotes progeny viral release
Amantadine is DOC for prophylaxis and treatment
It blocks viral penetration/uncoating,
it also can release dopamine from intact nerve terminal...is used forr
parkinson's disease
19.CC
Oculomotor nerveIII......eye movement(SR,IR,MR,IO)...pupillary constriction
,accommodation,eyelide opening(Levator Palpebrae)
20.BB
Allergic Rhinitis is defined as inflammation of the nasal membranes and is
characterized by a symptom complex that consists of any combination of the
following: sneezing, nasal congestion, nasal itching, and rhinorrhea.The
eyes, ears, sinuses, and throat can also be involved. Allergic rhinitis is the
most common cause of rhinitis.
Exposure to certain foreign proteins leads to allergic sensitization, which is
characterized by the production of specific IgE directed against these
proteins. This specific IgE coats the surface of mast cells, which are present
in the nasal mucosa. When the specific protein (eg, a specific pollen grain) is
inhaled into the nose, it can bind to the IgE on the mast cells, leading to
immediate and delayed release of a number of mediators.
The mediators that are immediately released include histamine, tryptase,
chymase, kinins, and heparin. The mast cells quickly synthesize other
mediators, including leukotrienes and prostaglandin D2. These mediators,
via various interactions, ultimately lead to the symptoms of rhinorrhea .
A pet that is isolated to one area of the house may still cause significant
problems for an person with allergic rhinitis because the dander can be
airborne and move to any part of the home through the ventilation system.

21.BB
Most humans are exposed to CMV in their lifetime, but typically only
individuals with weakened immune systems become ill from CMV infection.
Usually, CMV produces no symptoms. However, serious CMV infections can
occur in people with weakened immune systems due to AIDS, organ
transplants, bone marrow transplant, chemotherapy, or medicines that
suppress the immune system.
A CMV infection may affect different parts of the body. Infections include:
CMV esophagitis (infection of the esophagus)
CMV gastroenteritis (infection of the stomach or intestines)
CMV retinitis (infection of the eye)
CMV pneumonia (infection of the lung)
Mononucleosis-like illness

22.DD
ITs tricuspid Valve. The other is mitral valve. Its too low for aortic valve and
you dont do pulmonary artery replacement in adults.
So even by exclusion its tricuspid

23.FF
A Meckel's diverticulum, a true congenital diverticulum, is a small bulge in
the small intestine present at birth. It is a vestigial remnant of the
omphalomesenteric duct (also called the vitelline duct or yolk stalk), and is
the most frequent malformation of the gastrointestinal tract
Meckel's diverticulum is located in the distal ileum, usually within about 60100 cm (2 feet) of the ileocecal valve
The most common presenting symptom is painless rectal bleeding such as
melena-like black offensive stools, followed by intestinal obstruction,
volvulus and intussusception. Occasionally, Meckel's diverticulitis may
present with all the features of acute appendicitis. Also, severe pain in the
upper abdomen is experienced by the patient along with bloating of the
stomach region. At times, the symptoms are so painful such that they may
cause sleepless nights with extreme pain in the abdominal area.
Most of the time, bleeding occurs without warning and stops spontaneously.
The symptoms can be extremely painful, often mistaken as just belly pain
resulting from not eating or constipation.

24.BB
Because dual infection is common, patients diagnosed with either Gonorrhea
or Chlamydia
should receive empiric treatment for both infections, unless the other
infection has been ruled out.
Treatment of Chlamydia:
Azithromycin
Doxycycline
Treatment of Gonorrhea:
Ceftriaxone
Cefixime
25 Steroid-induced osteoporosis (SIOP) is osteoporosis arising due
to use of glucocorticoids - analogous to Cushing's syndrome and involving
mainly the axial skeleton. The synthetic glucocorticoid prescription drug
prednisone is a main candidate after prolonged intake. Some professional
guidelines recommend prophylaxis in patients who take the equivalent of
more than 30 mg hydrocortisone (7.5 mg of prednisolone), especially when
this is in excess of three months. Alternate day use may not prevent this
complication.
Mechanisms of SIOP include:
Direct inhibition of osteoblast function
Direct enhancement of bone resorption
Inhibition of gastrointestinal calcium absorption
Increased urine calcium loss
Inhibition of sex steroids

26.DD
Chronic exposure to relatively low levels of carbon monoxide may cause
persistent headaches, lightheadedness, depression, confusion, memory loss,
nausea and vomiting.
It is unknown whether low-level chronic exposure may cause permanent
neurological damage. Typically, upon removal from exposure to carbon
monoxide, symptoms usually resolve themselves, unless there has been an
episode of severe acute poisoning
27.CC
Although the kidneys receive nearly a quarter of the cardiac output and
extract relatively little oxygen, there is a marked discrepancy between
cortical and medullary blood flow and oxygen delivery and consumption.
In the medulla, blood flow and oxygen supply are restricted by a
tubulovascular anatomy specifically designed for urinary concentration.
28..Answer should be DD..
Characteristic of rapidly redistributed drug. At first the drug is rapidly
redistributed so the plasma level falls suddenly. This is followed by normal
elimination which causes the characteristic change in the slope..
29.DD
Classic galactosemia...due to Galactose-1-phosphate Uridyltransferase
Smptos:failure of thrive, Jaundice, hepatomegaly, cataracts and mental
retardation
Galactokinase deficiency....Galactitol accumulates if galactose is present in
diet.....mild condition

30.CC
retroviruses (HIV and HTLV) have reverse transcriptase
31.DD

http://en.wikipedia.org/wiki/File:Gray784.png
32.CC
At a single locus,only 1 allele is active,the other is
inactive(imprinted/inactivated by methylation)...deletion of active
allele......>disease

33.CC
Vascular endothelial growth factor (VEGF) is a signal protein produced by
cells that stimulates vasculogenesis and angiogenesis. It is part of the
system that restores the oxygen supply to tissues when blood circulation is
inadequate.
( P O2 VEGF)
34.CC
AML-M3...High circulating myeloblasts on peripheral smear
Auer bodies...Peroxide positive cytoplasmic inclusion in granulocytes and
myeloblasts
Treatment of AML M3 can release Auer rods..........>DIC
35.DD
Secondary Hyperparathyroidism.....due to chronic renal failure...causes
secondary hyperplasia of parathyroid due to dec Calcium ion absorption and
inc phosphorous
HYpocalcemia,Hyperphosphatemia,high Alkaline phosphatase,high PTH
36.BB
The cold agglutinins test is used to confirm the diagnosis of certain diseases
that stimulate the body to produce cold agglutinins.
mycoplasma pneumonia, a form of atypical bacterial pneumonia, and is
related to cold agglutinin disease.
This species lacks a peptidoglycan cell wall. Instead, it has a cell membrane
that incorporates sterol compounds, similar to eukaryotic cells. It obtains
these sterols from the host serum, allowing it to retain a simple structure.
Lacking a cell wall, these organisms are resistant to the effects of penicillins
and other beta-lactam antibiotics, which act by disrupting the bacterial cell
wall. Antibiotics with activity against these organisms include certain
macrolides (erythromycin, azithromycin, clarithromycin), fluoroquinolones
and their derivatives (e.g., ciprofloxacin, levofloxacin), and tetracyclines
(e.g., doxycycline).

37.CC
Persons with component deficiencies in the final common complement
pathway (C3,C5-C9) are more susceptible to N. meningitidis infection than
complement-satisfactory persons, and it was estimated that the risk of
infection is 7000 times higher in such individuals. In addition, complement
component-deficient population frequently experience frequent
meningococcal disease.
38.EE
Menopause is a term used to describe the permanent cessation of the
primary functions of the human ovaries.
It is the result of the eventual depletion of almost all of the oocytes and
ovarian follicles in the ovaries. This causes an increase in circulating follicle
stimulating hormone (FSH) and luteinizing hormone (LH) levels because
there are a decreased number of oocytes and follicles responding to these
hormones and producing estrogen.
This decrease in the production of estrogen leads to the perimenopausal
symptoms of hot flashes, insomnia and mood changes. Long term effects
may include osteoporosis and vaginal atrophy.
39.AA
Alprazolam is a potent short-acting drug of the benzodiazepine class.
It is primarily used to treat moderate to severe anxiety disorders (e.g., social
anxiety disorder) and panic attacks, and is used as an adjunctive treatment
for anxiety associated with moderate depression.
40.EE
Cleft lip...failure of fusion of the maxillary and medial nasal
process ...formation of primary palate
Cleft palate.....failure of fusion of the lateral palatine processes ,the nasal
septum, and/or the median palatine process......formation of secondary
palate

41.DD
Drug X is an alpha1 blocker
NE.....alpha1>alpha2>beta1.......has high alpha1 agonist power.....after
using drug X ......force dec. from 15.1.....6.8
Isoproterenol.....is a nonselective beta agonist........after using drug
X.....there is no significant change
Phenylephrine.....is an alpha agonist...alpha1>alpha2......after using drug
X.....there is a significant change....dec. force from 13.....to....5.5
So drug X must be an alpha 1 antagonist......between choices Prazosin is an
alpha 1 antagonist
42.DD
43.DD
Failure of Fusion of IVS with Endocardial cushions.
the difference O2 saturation% between right atrium ..75%....and right
ventricle... 83% shows there is a opening/shunt between two ventricles that
causes to move oxygenated blood from left to right ventricle....so
VSD....Holosystolic ,harsh sounding murmur; loudest at tricuspid area
44.CC
Secoundary hyperparathyroidism...look at Q35
PO4+, Ca2+, PTH
45.DD
Rheumatoid arthritis....Autoimmune inflammatory disease affecting synovial
joints with pannus formation in joint....MCP and PIP.......****no
DIP...Osteoarthritis***
Subcutaneous rheumatoid nodules, ulnar deviation, subluxation
Baker's cyst...Behind the knee nodules
80% have Positive Rheumatic factor...anti-antiG antibody
strong association with....HLA-DR4
clinical presentation.....morning stiffness improving with use, symmetric
joint involvement.. and systemic
symptoms...tempreture,fatigue,pleuritis,pericarditis
46 E
Right Main Lobe Foreign Body Obstruction

47 B Brother, if the new born Male low threshold mean more susceptible to
have the disease it's tricky Q
Males are more commonly affected than females, with firstborn males
affected about four times as often, and there is a genetic predisposition for
the disease.[2] It is commonly associated with people of Jewish ancestry,
and has multifactorial inheritance patterns.[3] Pyloric stenosis is more
common in whites than Hispanics, African Americans, or Asians.
http://en.wikipedia.org/wiki/Pyloric_stenosis
I think A...because the chance of getting of this abnormality in males more
than females..so if this Patient is a female the chance of her brother is much
more than other ones

Male Lower threshhold of liability = easily get Dz=


Greatest risk: Brother, if Newborn Female
b/c Female unlikely, so if get that then male very likely!
48.EE
The thymus and parathyroid, both derivatives of the 3rd pharyngeal pouches
49.CC
Bosentan is a dual endothelin receptor antagonist used in the treatment of
pulmonary artery hypertension (PAH).
Bosentan is a competitive antagonist of endothelin-1 at the endothelin-A
(ET-A) and endothelin-B (ET-B) receptors. Under normal conditions,
endothelin-1 binding of ET-A or ET-B receptors causes pulmonary
vasoconstriction. By blocking this interaction, bosentan decreases pulmonary
vascular resistance. Bosentan has a slightly higher affinity for ET-A than ETB.

50-D

Common variable hypogammaglobulinemia, unknown molecular


defect, onsets in late teens, early tewnties, B cells presents in peripheral
blood, immunoglobulin levels decreased with time,; increased autoimmunity,
the albumin can be normal or low, but the gamma globulin always is low,
this curve is the same for Lymphoproliferative disorders, inflammatory bowel
disease, congenital immunodeficiencies

BLOCK 4
1.CC
Antiphospholipid antibody testing may be ordered when a patient has
symptoms suggestive of a thrombotic episode, such as pain and swelling in
the extremities, shortness of breath, and headaches. It also may be ordered
when a woman has had recurrent miscarriages and/or as a follow-up to a
prolonged PTT test.
Cardiolipin antibodies (IgG, IgM, and sometimes IgA) are frequently ordered
as they are the most common antiphospholipids. If a patient has a prolonged
PTT test, further lupus anticoagulant testing is usually indicated
If the tests indicate the presence of the lupus anticoagulant and it persists
when retested, then it is likely that the patient is positive for the lupus
anticoagulant. Patients who have one or more antiphospholipid antibodies
and those that are diagnosed with antiphospholipid syndrome have an
increased risk of having recurrent thrombotic episodes, recurrent
miscarriages, and thrombocytopenia.
Occasionally, antiphospholipid testing may be ordered to help determine the
cause of a positive VDRL/RPR test for syphilis. The reagents used to test for
syphilis contain phospholipids and can cause a false positive result in
patients with antiphospholipid antibodies
2.EE
PD, also MDD Treated with SSRI (Fluoxetine). Don't Give __ FOR PD:
Selegiline (b/c MAO I = Serotonin Syn)
MAO inhibitors are contraindicated with SSRI's..
May cause serotonin syndrome due to excess synaptic serotonin..
S/S-diarrhea, flushing, muscle spasm, etc.
3.BB
Cohort study by concept is prospective, the patient are follow forward in
time, the population should be follow long enough for incidence to appear,
according to the question there is no decreased between both group ---MI
and placebo group----the study get the information in retrospective
condition. C, D, E discarded to precoz, no enough time with the study to
place en practice----- A NO because cross sectional is used to determine
prevalence or to establish etiology

4.DD

Syphilis is a sexually transmitted disease caused by the spirochetal bacteria


Treponema pallidum subspecies pallidum. The primary route of transmission
of syphilis is through sexual contact however it may also be transmitted from
mother to fetus during pregnancy or at birth resulting in congenital syphilis.
The signs and symptoms of syphilis vary depending on which of the four
stages it presents in (primary, secondary, latent, and tertiary). The primary
stage typically presents with a single chancre, secondary syphilis with a
diffuse rash, latent with little to no symptoms, and tertiary with gummas,
neurological, or cardiac symptoms. Diagnosis is usually via blood tests. It
can be effectively treated with antibiotics, specifically intramuscular penicillin
G.

5.EE
Type II cells are responsible for the production and secretion of surfactant
(the majority of which are dipalmitoylphosphatidylcholine), a group of
phospholipids that reduce the alveolar surface tension. Surfactant
phospholipids are stored in Type II pneumocytes in lamellar bodies, which
are specialized vesicles. Release of surfactant in lamellar bodies occurs from
an infant's first breath onwards.
Type II pneumocytes can replicate in the alveoli and will replicate to replace
damaged Type I pneumocytes.

6.CC

Body store iron deficiency is diagnosed by diagnostic tests as a low serum


ferritin, a low serum iron level, an elevated serum transferrin and a high
total iron binding capacity (TIBC). A low serum ferritin is the most sensitive
lab test for iron deficiency anemia
Serum iron levels (i.e., iron not part of the hemoglobin in red cells) may be
measured directly in the blood, but these levels increase immediately with
iron supplementation.
The diagnosis of IDA requires that a patient be anemic and show laboratory
evidence of iron deficiency. Red blood cells in IDA are usually described as
being microcytic (i.e., mean corpuscular volume less than 80 m3 [80 fL])
and hypochromic, however the manifestation of iron deficiency occurs in
several stages.

7.DD
Hydrocholorothiazide ......Hypercalcemia
http://courses.washington.edu/bonephys/hypercalU/opthiazide.html
8.DD
IgG is the only Ig that transports through the placenta

9. C
Pneumothorax (plural pneumothoraces) is a collection of air or gas in the
pleural cavity of the chest between the lung and the chest wall. It may occur
spontaneously in people without chronic lung conditions ("primary") as well
as in those with lung disease ("secondary"), and many pneumothoraces
occur after physical trauma to the chest, blast injury, or as a complication of
medical treatment.[1][2]
The symptoms of a pneumothorax are determined by the size of the air leak
and the speed by which it occurs; they may include chest pain in most cases
and shortness of breath in many. The diagnosis can be made by physical
examination in severe cases but usually requires a chest X-ray or computed
tomography (CT scan) in milder forms. In a small proportion, the
pneumothorax leads to severe oxygen shortage and low blood pressure,
progressing to cardiac arrest unless treated; this situation is termed tension
pneumothorax

10. B
Hypertrophic cardiomyopathy is a disease of the myocardium (the muscle of
the heart) in which a portion of the myocardium is hypertrophied (thickened)
without any obvious cause.[1][2][3][4][5][6] It is perhaps most well-known
as a leading cause of sudden cardiac death in young athletes.[7] The
occurrence of hypertrophic cardiomyopathy is a significant cause of sudden
unexpected cardiac death in any age group and as a cause of disabling
cardiac symptoms. Younger people are likely to have a more severe form of
hypertrophic cardiomyopathy
HCM is frequently asymptomatic until sudden cardiac death, and for this
reason some suggest routinely screening certain populations for this disease.
[8]
A cardiomyopathy is a primary disease that affects the muscle of the heart.
With hypertrophic cardiomyopathy (HCM), the sarcomeres (contractile
elements) in the heart replicate causing heart muscle cells to increase in
size, which results in the thickening of the heart muscle. In addition, the
normal alignment of muscle cells is disrupted, a phenomenon known as
myocardial disarray. HCM also causes disruptions of the electrical functions
of the heart. HCM is most commonly due to a mutation in one of 9
sarcomeric genes that results in a mutated protein in the sarcomere, the
primary component of the myocyte (the muscle cell of the heart).
Myosin heavy chain mutations are associated with development of familial
hypertrophic cardiomyopathy.
About 50-60% of patients with a high index of clinical suspicion for HCM will
have a mutation identified in at least 1 of 9 sarcomeric genes. Approximately
45% of these mutations occur in the myosin heavy chain gene on
chromosome 14 q11.2-3, while approximately 35% involve the cardiac
myosin binding protein C gene. Since HCM is typically an autosomal
dominant trait, children of an HCM parent have 50% chance of inheriting the
disease-causing mutation. Whenever a mutation is identified through genetic
testing, family-specific genetic testing can be used to identify relatives atrisk for the disease (HCM Genetic Testing Overview). In individuals without a
family history of HCM, the most common cause of the disease is a de novo
mutation of the gene that produces the -myosin heavy chain.
11. B
Dmg Blood and Nerve supply to Erectile Tissue!
Erectile dysfunction (ED, "male impotence") is sexual dysfunction
characterized by the inability to develop or maintain an erection of the penis
during sexual performance.
A study in 2002 found that ED can also be associated with bicycling. The
number of hours on a bike and/or the pressure on the penis from the saddle
of an upright bicycle is directly related to erectile dysfunction

Causes
Drugs (anti-depressants (SSRIs) and nicotine are most common)
Neurogenic disorders (spinal cord and brain injuries, nerve disorders
such as Parkinson's disease, Alzheimer's disease, multiple sclerosis,
and stroke[4])
Cavernosal disorders (Peyronie's disease[5])
Psychological causes: performance anxiety, stress, mental disorders
(clinical depression, schizophrenia, substance abuse, panic disorder,
generalized anxiety disorder, personality disorders or traits[6]),
psychological problems, negative feelings.[7][not in citation given]
Surgery (radiation therapy, surgery of the colon, prostate, bladder, or
rectum may damage the nerves and blood vessels involved in erection.
Prostate and bladder cancer surgery often require removing tissue and
nerves surrounding a tumor, which increases the risk for impotence [8])
Ageing. It is four times higher in men in their 60s than in men in their
40s.[9]
Kidney failure
Diseases such as diabetes and multiple sclerosis (MS). While these two
causes have not been proven theyre likely suspects as they cause
issues with both the blood flow and nervous systems.
Lifestyle: smoking is a key cause of erectile dysfunction. [10][11] Smoking
causes impotence because it promotes arterial narrowing

12. D
Type IV Hypersensitivity: Delayed type ( 48 - 72 hrs), CD4 +TH1 cells
mediate, Activate macrophages, causes inflammation, common in chronic
intracellular inflammation. (Kaplan micro immuno book page 158).
13. D
Tumor Best Prog : Poorly differentiated carcinoma confined to mucosa

Stage 0 Tis N0 M0 Tis: Tumor confined to mucosa; cancer-in-situ


Stage I T1 N0 M0 T1: Tumor invades submucosa
Stage I T2 N0 M0 T2: Tumor invades muscularis propria
Stage II-A T3 N0 M0 T3: Tumor invades subserosa or beyond (without other
organs involved)
Stage II-B T4 N0 M0 T4: Tumor invades adjacent organs or perforates the
visceral peritoneum
Stage III-A T1-2 N1 M0 N1: Metastasis to 1 to 3 regional lymph nodes. T1 or
T2.
Stage III-B T3-4 N1 M0 N1: Metastasis to 1 to 3 regional lymph nodes. T3 or
T4.
Stage III-C any T, N2 M0 N2: Metastasis to 4 or more regional lymph nodes.
Any T.
Stage IV any T, any N, M1 M1: Distant metastases present. Any T, any N.
http://en.wikipedia.org/wiki/Colorectal_cancer#Staging
14. D
Posterior urethral valve (PUV) disorder is an obstructive developmental
anomaly in the urethra and genitourinary system of male newborns.[1] A
posterior urethral valve is an obstructing membrane in the posterior male
urethra as a result of abnormal in utero development. It is the most common
cause of bladder outlet obstruction in male newborns. The disorder varies in
degree, with mild cases followed conservatively. More severe cases can have
renal failure and even respiratory failure from lung underdevelopment as
result of low amniotic fluid volumes, requiring intensive care and close
monitoring.
Abdominal ultrasound is of some benefit, but not diagnostic. Features that
suggest posterior urethral valves are bilateral hydronephrosis, a thickened
bladder wall with thickened smooth muscle trabeculations, and bladder
diverticula.

15.BB

B.....100% sensitivity
C.....most accurate
D....100%specificity
16.DD
Hepatitis- Hep B. Direct dmg to hepatocytes by Cytotoxic T Lymphocytes.
Natural killer cells (or NK cells) are a type of cytotoxic lymphocyte that
constitute a major component of the innate immune system. NK cells play a
major role in the rejection of tumors and cells infected by viruses.
NK cells are cytotoxic; small granules in their cytoplasm contain proteins
such as perforin and proteases known as granzymes. Upon release in close
proximity to a cell slated for killing, perforin forms pores in the cell
membrane of the target cell, creating an aqueous channel through which the
granzymes and associated molecules can enter, inducing either apoptosis or
osmotic cell lysis.

17.CC

Ehler-Danlos syndrome...faulty collagen synthesis


causing:
Hyper extensible skin
tendency to bleed
hypermobile joint...may be associated with joint dislocation, berry aneurism
and organ rupture
Type III collagen is most frequently accented
Can be AR or AD

18.CC
Increased estrogen produces higher levels of thyroid-binding globulin, a
protein that transports thyroid hormone in the blood. These normal
hormonal changes can sometimes make thyroid function tests during
pregnancy difficult to interpret.
Low TSH levels may occur in a normal pregnancy, however, especially in the
first trimester. If TSH levels are low, another blood test is performed to
measure T4 and T3.
Elevated levels of free T4the portion of thyroid hormone not attached to
thyroid-binding proteinsconfirm the diagnosis. Rarely, free T4 levels are
normal in a woman with hyperthyroidism but T3 levels are high. Because of
normal pregnancy-related changes in thyroid function, test results must be
interpreted with caution.
19.BB
The ureter passes under the uterine artery in females and the ductus
deferens in the males in the pelvis. The gonadal arteries cross the ureter in
its middle 1/3 in the abdominal cavity where it overlies posas major.
In pelvic surgery, particularly during hysterectomy, the ureter is at risk when
the uterine artery is ligated and divided

20.AA
High levels of AFP may suggest that the developing baby has a neural tube
defect such as spina bifida or anencephaly. High levels of AFP may also
suggest defects with the esophagus or a failure of the baby's abdomen to
close.However, the most common reason for elevated AFP levels is
inaccurate dating of the pregnancy.
Low levels of AFP and abnormal levels of hCG and estriol may indicate that
the developing baby has Trisomy 21( Down syndrome), Trisomy 18 (Edwards
Syndrome) or another type of chromosome abnormality
21.CC
Baclophen is an agonist for the GABAB receptors. Its beneficial effects in
spasticity result from actions at spinal and supraspinal site.
Baclofen is widely used for the treatment of spastic movement disorders,
especially in instances of spinal cord injury, spastic diplegia cerebral palsy,
multiple sclerosis, amyotrophic lateral sclerosis , peripheral neuropathy and
trigeminal and glossopharyngeal neuralgias.
22.BB
pleiotropy
the production by a single gene of multiple phenotypic effects. The term is
often used to refer to a single gene defect that is expressed as problems in
multiple systems of the body, such as in osteogenesis imperfecta, where the
gene causes defects in several different systems that contain collagen.
23.DD
Tay-Sac Disease is caused by insufficient activity of an enzyme called
hexosaminidase A that catalyzes the biodegradation of fatty acid derivatives
known as gangliosides. Hexosaminidase A is a vital hydrolytic enzyme, found
in the lysosomes, that breaks down phospholipids.
When Hexosaminidase A is no longer functioning properly, the lipids
accumulate in the brain and interfere with normal biological processes. GM2
Gangliosides are made and biodegraded rapidly in early life as the brain
develops. Patients and carriers of TaySachs disease can be identified by a
simple blood test that measures hexosaminidase A activity.

24.DD
Anthrax is a disease caused by Bacillus anthracis, a spore-forming, Gram
positive, rod-shaped bacterium (Fig. 1). The lethality of the disease owes
itself to the bacterium's two principal virulence factors:
(i) the polyglutamic acid capsule, which is anti-phagocytic, and
(ii) the tripartite protein toxin, called anthrax toxin.
Anthrax toxin is a mixture of three protein components: (i) protective
antigen (PA), (ii) edema factor (EF), and (iii) lethal factor (LF).
25.EE
Gynecomastia is overdevelopment of the male breast. The glandular tissue
of the breast swells, usually in response to an excess of the female hormone
estrogen or a lack of testosterone, a male hormone. It occurs in babies, teen
boys, and older men.
26.AA
It is Sargramostim....GM-CSF
- For febrile neutropenia: Adults dosage is 510 g/kg IV or SC once daily is
sufficient to support the Absolute Neutrophil Count during an episode of
febrile neutropenia.
- For the treatment of congenital, idiopathic, and cyclic neutropenia and
hastens the recovery of marrow from neutropenia after cancer
chemotherapy
27.CC
Supraspinatus helps deltoid abduct arm...innervated by Axillary n.
Infraspinatus ..latealyrotate arms
Teres Minor.....adducts and laterally rotates arm
Subscapularis..medially rotates and adducts arm
28.BB

Rapid correction of hyponatremia causing Central Pontine myelinosis which is


characterized by acute paralysis, dysphagia, dysarthria and loss of
consciousness.
Wernicke encephalopathy is a syndrome characterized by ataxia,
ophthalmoplegia, confusion, and impairment of short-term memory. It is
caused by lesions in the medial thalamic nuclei, mammillary bodies,
periaqueductal and periventricular brainstem nuclei, and superior cerebellar
vermis, often resulting from inadequate intake or absorption of thiamine
(vitamin B1), especially in conjunction with carbohydrate ingestion.
It is most commonly correlated with prolonged alcohol, consumption
resulting in thiamine deficiency.
29.FF
Reovirus family.. no envelope...DS RNA ...10-12
segments ..1.Rotavirus(children diarrhea) and 2. Reovirus (Colorado tick
fever)
Rotavirus is number 1 cause of fetal diarrhea in children less than 2...nonbloody ,watery stool and ELISA+
30.DD
Edema is caused by...
1.inc. capillary pressure (inc. capillary pressure and heart failure)
2.low plasma proteins...(dec. plasma colloid osmotic pressure. nephrotic
syndrome ,liver failure)
3. inc. capillary permeability (toxin, infection, burns)
4.inc. interstitial fluid colloid osmotic pressure(lymphatic blockage)
31. CC

Right Phrenic nerve and i got it right. The mediastinal space which is present
between the parietal layer of serous pericardium and the fibrous pericardium
contains both the phrenic N and the vagus N.So it can be both phrenic and
vagus which are given in the options but i guessed it and i got it right.

The answer should be C Dx of this CT picture really big pericardial effusion


. Breast cancer is the second most common cause of malignant pericardial
effusions. Approximately one in four patients with malignant pericardial
effusions has breast cancer, and autopsy findings indicate that of all patients
with breast cancer, approximately one in four had malignant pericardial
effusion. Hematologic malignancies such as leukemia, Hodgkin disease, and
non-Hodgkin lymphoma make up the third largest group (15% of cases).
the primary.
Lung cancer, the most frequent source of neoplastic pericardial disease,
accounts for about one-third of cases.
PLEASE check this video:http://radiographics.rsna.org/content/21/2/439.full
On both sides the phrenic nerve runs posterior to the subclavian vein and
posterior to the internal thoracic artery as it enters the thorax.[1]
Both of these nerves supply motor fibers to the diaphragm and sensory
fibers to the fibrous pericardium, mediastinal pleura, and diaphragmatic
peritoneum.
Open to more explanations
32.AA
significant factor in AD is the greatly reduced presence of acetylcholine in
the cerebral cortex. Acetylcholine is necessary for cognitive function.
Alzheimer's disease (AD) is an irreversible, progressive disorder in which
brain cells (neurons) deteriorate, resulting in the loss of cognitive functions,
primarily memory, judgment and reasoning, movement coordination, and
pattern recognition.
In advanced stages of the disease, all memory and mental functioning may
be lost.
The cerebral cortex is an extremely convoluted and complicated structure
associated with the "higher" functions of the mindthought, reasoning,
sensation, and motion.
The hippocampus plays a crucial role in learning and in processing various
forms of information as long-term memory. Damage to the hippocampus
produces global amnesia

33.DD
Multi-infarct dementia, is one type of vascular dementia. Vascular dementia
is the second most common form of dementia after Alzheimer's disease (AD)
in older adults.
Multi-infarct dementia (MID) is thought to be an irreversible form of
dementia, and its onset is caused by a number of small strokes or
sometimes, one large stroke preceded or followed by other strokes.
Risk factors for vascular dementia include hypertension, smoking,
hypercholesterolemia, diabetes mellitus, and cardiovascular and
cerebrovascular disease
34.CC
The primary symptoms of IBS are abdominal pain or discomfort in
association with frequent diarrhea or constipation, a change in bowel
habits.There may also be urgency for bowel movements, a feeling of
incomplete evacuation (tenesmus), bloating or abdominal distention.
People with IBS more commonly than others have gastroesophageal reflux,
symptoms relating to the genitourinary system, chronic fatigue syndrome,
fibromyalgia, headache, backache and psychiatric symptoms such as
depression and anxiety. Some studies indicate that up to 60% of persons
with IBS also have a psychological disorder, typically anxiety or depression.
35.EE
Myelofibrosis, also known as myeloid metaplasia, is a disorder of the bone
marrow. It is currently classified as a myeloproliferative disease in which the
proliferation of an abnormal type of bone marrow stem cell results in
fibrosis, or the replacement of the marrow with collagenous connective
tissue fibers.
Abdominal fullness related to an enlarged spleen (splenomegaly).
Bone pain
Bruising and easy bleeding due to inadequate numbers of platelets
Fatigue
Increased susceptibility to infection, such as pneumonia or diarrhea
Pallor and shortness of breath while doing physical work due to anemia
Because of a high rate of cell turnover, hyperuricemia and gout may also
complicate the picture
The bone marrow is replaced by collagen fibrosis, impairing the patient's
ability to generate new blood cells resulting in a progressive pancytopenia

36. C
Mitochondria have their own genetic material, and the machinery to
manufacture their own RNAs and proteins.
Leigh's disease, also known as Subacute Necrotizing Encephalomyelopathy
(SNEM), is a rare neurometabolic disorder that affects the central nervous
system.
There is another form of this disease called the X-linked Leigh's syndrome
(OMIM 308930), which is a mutation in the oxidative phosphorylation
enzymes (which are on both the mtDNA and the nuclear DNA). The X-linked
Leigh's disease is a mutation of a gene encoding PDHA1, part of the
pyruvate dehydrogenase complex, located on the X chromosome
It is an inherited disorder that usually affects infants between the age of
three months and two years, but, in rare cases, teenagers and adults as
well. In the case of the disease, mutations in mitochondrial DNA (mtDNA) or
in nuclear DNA (gene SURF1[2] and some COX assembly factors) cause
degradation of motor skills and eventually death.
Mitochondria are an essential organelle in eukaryotic cells. Their function is
to convert the potential energy of glucose, amino acids, and fatty acids into
adenosine triphosphate (ATP). Mitochondria carry their own DNA, called
mitochondrial DNA [mtDNA]. The information stored in the mtDNA is used to
produce several of the enzymes essential to the production of ATP.
Mutations in the mtDNA that cause the mitochondria to fail, to function
improperly, a person is at risk for a number of disorders, including Leigh's
disease. In the case of Leigh's disease, crucial cells in the brain stem have
mutated mtDNA, creating poorly functioning mitochondria. This causes a
chronic lack of energy in the cells, which, in turn, affects the central nervous
system and inhibits motor functions.
http://en.wikipedia.org/wiki/Leigh's_disease

37. B
Mebendazole or MBZ is a benzimidazole drug developed by Janssen
Pharmaceutica and marketed as Vermox, Ovex, Antiox, and Pripsen. It is
used to treat infestations by worms including pinworms, roundworms,
tapeworms, hookworms, and whipworms
Mechanism of action
Mebendazole is thought to work by selectively inhibiting the synthesis of
microtubules in parasitic worms, and by destroying extant cytoplasmic
microtubes in their intestinal cells: thereby blocking the uptake of glucose
and other nutrients, resulting in the gradual immobilization and eventual
death of the helminths. The drug is a highly effective broad spectrum
antihelminitic indicated for the treatment of nematode infestations, including
round worm, whip worm, thread worm, and hook worm. It is poorly
absorbed and has no systemic effects.
38. C
39. A

Acute pyelonephritis: Affects cortex with relative sparing of glomeruli


/vessels white cells cast in urine are classic. Present fever, CVA tenderness,
nausea and vomiting. Acute pyelonephritis is characterized by neutrophilic
infiltration and abscess formation within the renal interstitium. Abscess may
rupture, introduction collections of white cells to the tubular lumen.
40. A

x
Middle cerebral artery syndrome is a condition whereby the blood supply
from the middle cerebral artery (MCA) is restricted, leading to a reduction of
the function of the portions of the brain supplied by that vessel: the lateral
aspects of frontal, temporal and parietal lobes, the corona radiata, globus
pallidus, caudate and putamen. The MCA is the most common site for the
occurrence of ischemic stroke
Signs and Symptoms
1. Hemiparesis or hemiplegia of the lower half of the contralateral face[2]
2. Hemiparesis or hemiplegia of the contralateral upper and lower
extremities*[2]
3. Sensory loss of the contralateral face, arm and leg*[2]
4. Ataxia of contralateral extremities*[1]

5. Speech impairments/aphasia: Broca's, Wernicke's or Global aphasia as a


result of a dominant hemisphere lesion (usually the left brain)[1][2][3]
6. Perceptual deficits: hemispatial neglect, anosognosia, apraxia, and spatial
disorganization as a result of a non-dominant hemisphere lesion (usually the
right brain)[1][3]
7. Visual disorders: deviation conjuge, a gaze preference towards the side
of the lesion; contralateral homonymous hemianopsia.
41. B
Diabetic ketoacidosis (DKA) is a potentially life-threatening complication in
patients with diabetes mellitus. It happens predominantly in those with type
1 diabetes, but it can occur in those with type 2 diabetes under certain
circumstances. DKA results from a shortage of insulin; in response the body
switches to burning fatty acids and producing acidic ketone bodies that
cause most of the symptoms and complications.
Diabetes mellitus type 1 (Type 1 diabetes, IDDM, or, formerly, juvenile
diabetes) is a form of diabetes mellitus that results from autoimmune
destruction of insulin-producing beta cells of the pancreas.[2] The
subsequent lack of insulin leads to increased blood and urine glucose. The
classical symptoms are polyuria (frequent urination), polydipsia (increased
thirst), polyphagia (increased hunger), and weight loss.
Pathophysiology
The pathophysiology in diabetes type I is basically a destruction of beta cells
in the pancreas, regardless of which risk factors or causative entities have
been present.
Individual risk factors can have separate pathophysiological processes to, in
turn, cause this beta cell destruction. Still, a process that appears to be
common to most risk factors is an autoimmune response towards beta cells,
involving an expansion of autoreactive CD4+ and CD8+ T helper cells,
autoantibody-producing B cells and activation of the innate immune system.
42. C
In contraction, the wound is made smaller by the action of myofibroblasts,
which establish a grip on the wound edges and contract themselves using a
mechanism similar to that in smooth muscle cells. When the cells' roles are
close to complete, unneeded cells undergo apoptosis.
If contraction continues for too long, it can lead to disfigurement and loss of
function.

43. B.
Anorexia nerviosa: excessive dieting +/- purging intense fear of gaining
weight , body imagen distortion, and exercise , leading to body weight <
85% below ideal body weight. Associated with body density.severe weight
loss, metatarsal stress fractures, amenorrhea ( GnRH, LH, FSH,
estrogen), anemia, and electrolyte disturbances. Sen primarily in adolescent
girls. Commonly coexist with depression. ( FA 2010 page 447)
44. D
Vitamine B12 defficiency: Macrocytic , megaloblastic anemia,
hypersegmented PMN, neurologic symptoms ( paresthesias, subacuted
combined degeneration) due to anormal myelin. Prolonged deficiency leads
to irreversible nervous system damage
45. E
Standard deviation is a widely used measurement of variability or diversity
used in statistics and probability theory. It shows how much variation or
"dispersion" there is from the average (mean, or expected value). A low
standard deviation indicates that the data points tend to be very close to the
mean, whereas high standard deviation indicates that the data are spread
out over a large range of values.
46. A
Calcitonin: Source: Parafollicular cells ( C cells ) of thyroid. Acts to bone
reabsorption. Regulation: serum Ca causes calcitonin secretion. ( FA 2010
page 289)

47. A
48. D
Phenylketonuria: Due to phenylalanine hydroxylase, or
tetrahydrobiopterin cofactor. Tyrosine becomes essential. Findings: mental
retardation, growth retardation, seizures, fair skin, eczema, musty body
odor. Treatment: phenylalanine ( contained in aspartame, eg, Nutrasweet)
and thyrosine in diet. ( FA page 107)
49. D
Methrotexate: mechanism of action
The similar structure of dihydrofolic acid (top) and methotrexate (top)
suggests that methotrexate is a competitive inhibitor
Methotrexate competitively inhibits dihydrofolate reductase (DHFR), an
enzyme that participates in the tetrahydrofolate synthesis.[4] The affinity of
methotrexate for DHFR is about one thousand-fold that of folate. DHFR
catalyses the conversion of dihydrofolate to the active tetrahydrofolate. Folic
acid is needed for the de novo synthesis of the nucleoside thymidine,
required for DNA synthesis. Also, folate is needed for purine base synthesis,
so all purine synthesis will be inhibited. Methotrexate, therefore, inhibits the
synthesis of DNA, RNA, thymidylates, and proteins.
Methotrexate acts specifically during DNA and RNA synthesis, and thus it is
cytotoxic during the S-phase of the cell cycle. Logically, it therefore has a
greater toxic effect on rapidly dividing cells (such as malignant and myeloid
cells, and gastrointestinal and oral mucosa), which replicate their DNA more
frequently, and thus inhibits the growth and proliferation of these
noncancerous cells, as well as causing the side effects listed below. Facing a
scarcity of dTMP, rapidly dividing cancerous cells undergo cell death via
thymineless death.
Thymidylate synthetase (EC 2.1.1.45)[1] is the enzyme used to generate
thymidine monophosphate (dTMP), which is subsequently phosphorylated to
thymidine triphosphate for use in DNA synthesis and repair.
5,10-methylenetetrahydrofolate + dUMP dihydrofolate + dTMP
By means of reductive methylation, deoxyuridine monophosphate (dUMP)
and N5,N10-methylene tetrahydrofolate are together used to form dTMP,
yielding dihydrofolate as a secondary product.
50. E
Carotid Massage: afferent activity, Parasympathetic, sympathetic
activity, HR, , BP

Você também pode gostar